Vous êtes sur la page 1sur 38

MINISTERIO DE SANIDAD, SERVICIOS SOCIALES E IGUALDAD

PRUEBAS SELECTIVAS 2011


CUADERNO DE EXAMEN

MDICOS -VERSIN 0 -

ADVERTENCIA IMPORTANTE
ANTES DE COMENZAR SU EXAMEN, LEA ATENTAMENTE LAS SIGUIENTES

INSTRUCCIONES
1. MUY IMPORTANTE : Compruebe que este Cuaderno de Examen lleva todas sus pginas y no
tiene defectos de impresin. Si detecta alguna anomala, pida otro Cuaderno de Examen a la
Mesa. Realice esta operacin al principio, pues si tiene que cambiar el cuaderno de examen
posteriormente, se le facilitar una versin "0", que no coincide con su versin personal en la
colocacin de preguntas y no dispondr de tiempo adicional.
2. El cuestionario se compone de 225 preguntas ms 10 de reserva. Tenga en cuenta que hay 30
preguntas que estn ligadas a una imagen. Todas las imgenes estn colocadas en las tres
hojas finales de este cuestionario.
3. Compruebe que el nmero de versin de examen que figura en su Hoja de Respuestas, coincide con el nmero de versin que figura en el cuestionario. Compruebe tambin el resto de
sus datos identificativos
4. La Hoja de Respuestas est nominalizada. Se compone de tres ejemplares en papel autocopiativo que deben colocarse correctamente para permitir la impresin de las contestaciones en
todos ellos. Recuerde que debe firmar esta Hoja.
5. Compruebe que la respuesta que va a sealar en la Hoja de Respuestas corresponde al nmero de pregunta del cuestionario. Slo se valoran las respuestas marcadas en la Hoja de Respuestas, siempre que se tengan en cuenta las instrucciones contenidas en la misma.
6. Si inutiliza su Hoja de Respuestas pida un nuevo juego de repuesto a la Mesa de Examen y no
olvide consignar sus datos personales.
7. Recuerde que el tiempo de realizacin de este ejercicio es de cinco horas improrrogables y
que estn prohibidos el uso de calculadoras (excepto en Radiofsicos) y la utilizacin de telfonos mviles, o de cualquier otro dispositivo con capacidad de almacenamiento de informacin
o posibilidad de comunicacin mediante voz o datos.
8. Podr retirar su Cuaderno de Examen una vez finalizado el ejercicio y hayan sido recogidas las
Hojas de Respuesta por la Mesa.

-1-

1.

Pregunta vinculada a la imagen n1

gen. Cules son los hallazgos de la ecografa?

Paciente de 55 aos de edad ex-fumador y ex


enlico. Hemorragia digestiva alta por ulcus
antral que requiri transfusin sangunea. Cirrosis heptica con serologas virales positivas
(VHC+). Child-Pugh de 6 puntos. La imagen
muestra dos secciones axiales en fase arterial y
tarda del TC realizado a la paciente. En relacin a los hallazgos de la prueba radiolgica cul
de las siguientes afirmaciones es cierta?

1. Vescula biliar distendida con litiasis en su


interior.
2. Engrosamiento de la pared de la vescula
biliar.
3. Inflamacin hipoecoica de la cabeza del
pncreas.
4. Litiasis y dilatacin del coldoco.
5. Aumento de calibre de la vena cava.

1. Se observa una lesin focal hipervascular en


fase arterial compatible con pseudoaneurisma
de la arteria heptica.
2. Se observa un ndulo hipervascular en fase
arterial con lavado en fase venosa compatible
con carcinoma hepatocelular.
3. Se observa una lesin focal hipovascular
compatible con colangiocarcinoma perifrico.
4. Se trata de una lesin qustica. Probable
quiste hidatdico.
5. Se observa una gran trombosis portal tumoral.

2.

4.

Cul es la actitud ms recomendable en este


momento?
1. Ciruga abierta urgente de la va biliar.
2. Antibioterapia y medidas sintomticas.
3. Colangiografa retrgrada endoscpica con
papilotoma.
4. Plantear colecistectoma diferida.
5. Solicitar una ecoendoscopia para valorar el
pncreas.

Pregunta vinculada a la imagen n1

5.

En el caso anterior qu tratamiento, entre los


siguientes, propondra en primer lugar?
1.
2.
3.
4.
5.

3.

Pregunta vinculada a la imagen n2

Pregunta vinculada a la imagen n3


Hombre de 62 aos, exfumador, colecistectomizado, sin otros antecedentes de inters. Acude al
servicio de urgencias por haber sufrido un sncope mientras caminaba: se diriga a su centro
de trabajo cuando, sin sntomas previos, sufri
prdida de conciencia con cada al suelo. Se recuper de forma espontnea en 2-3 minutos, sin
amnesia ni relajacin de esfnteres, si bien desde
entonces se encuentra mareado. Cuando llega a
urgencias tiene TA 155/85 mm Hg, 50 lpm, una
glucemia capilar de 135 mg/dl y el electrocardiograma que se adjunta. Seale la respuesta CORRECTA respecto al diagnstico del paciente:

Trasplante heptico.
Radioterapia externa.
Ablacin con radiofrecuencia.
Embolizacin arterial.
Quimioterapia sistmica.

Pregunta vinculada a la imagen n2


Hombre de 69 aos de edad con antecedentes de
diabetes mellitus y consumo moderado de alcohol desde haca aos, que acude a Urgencias por
dolor abdominal tipo clico, de 7 das de evolucin en hemiabdomen superior, ms localizado
en hipocondrio derecho. Asocia prurito generalizado que le dificulta el sueo y orinas oscuras en
los ltimos dos das. En la exploracin fsica
destacaba TA 130/61, FC 102 lpm, T 36.8C. El
abdomen estaba blando, depresible, doloroso a
la palpacin profunda y sensacin de masa en el
hipocondrio derecho. Los anlisis de sangre
muestran protena C reactiva 12.0 mg/dL, Glucosa 115 mg/dL, Urea 45 mg/dL, Creatinina 0.72
mg/dL, Bilirrubina total 8.45 mg/dL, electrolitos
normales, ALT (GPT plasma) 45 U/L, GGT 112
U/L, LDH 110 U/L, Lipasa 16 U/L. Leucocitos
9.3 10E3/l, hemates 4.08 10E6/l, hemoglobina
12.3 g/dl, hematocrito 35.9 %, VCM 87.9 fl, recuento de plaquetas 217 10E3/l, (N 69.3 %, L
26.2 %, M 4.5 %, Eo 0.0 %, B 0.0 %). Se efect
ecografa abdominal que se muestra en la ima-

1.
2.
3.
4.
5.

6.

Bloqueo AV de primer grado.


Bloqueo AV de segundo grado Mobitz 1.
Bloqueo AV de segundo grado Mobitz 2.
Bloqueo AV de tercer grado.
Ritmo sinusal con extrasistolia auricular.

Pregunta vinculada a la imagen n3


En referencia al paciente del caso clnico anterior, seale cul es la opcin teraputica ms
apropiada.
1. El paciente ha de ser ingresado para implantarle un marcapasos definitivo.
2. Se trata de un sncope vasovagal y el bloqueo
es de bajo grado por lo que puede ser dado de
alta para seguimiento y control ambulatorio.

-2-

3. Para el manejo de este paciente es indispensable la realizacin de una coronariografa


con vistas a descartar sustrato coronario que
justifique su clnica.
4. Si la extrasistolia auricular es muy sintomtica se puede pautar tratamiento con betabloqueantes.
5. Necesitamos una TC craneal para descartar
patologa neurolgica que justifique su clnica.

7.

formina. Presenta tos y expectoracin habitual


junto con una disnea basal a esfuerzos moderados/pequeos. Hace cinco meses comienza con
un cuadro de dolor, parestesias y disestesias en
ambas extremidades superiores, junto con una
alteracin de la sensibilidad vibratoria y posicional, con reflejos normales y sin debilidad o
atrofia muscular. En las ltimas tres semanas
refiere disfona, incremento de la tos y de la
disnea habituales y expectoracin amarillenta.
En los ltimos cinco das tiene cefaleas frecuentes, mareos y vmitos, motivo por el que acude a
urgencias, donde se objetiva una temperatura
de 37,8C; presin arterial de 140/85 mm Hg y
frecuencia cardiaca de 90 lpm sin taquipnea
significativa en reposo. El hemograma es normal
y en la bioqumica se observan un discreto aumento de las transaminasas y de la fosfatasa
alcalina. Se comprueba, adems, una hiponatremia de 119 mEq/L, con cloro de 88 mEq/L,
potasio de 3,9 mEq/L, osmolaridad plasmtica
de 255 mOsm/Kg, osmolaridad urinaria de 550
mOsm/kg y excrecin urinaria de sodio de 65
mEq/L. En el Servicio de Urgencias se le realiza
una radiografa de trax (figura). Cul es la
prueba diagnstica ms eficaz para establecer
un diagnstico definitivo en este caso?

Pregunta vinculada a la imagen n4


Un hombre de 67 aos no fumador acude a consulta por tos seca y disnea progresiva desde hace
ao y medio. A la exploracin presenta crepitantes tipo velcro en bases y acropaquias. Una gasometra arterial basal muestra pH 7.36, pCO2
34 mm Hg, pO2 62 mm Hg. Los valores de una
espirometra son FVC: 1550 ml (48% del valor
terico), FEV1 1320 ml (54% del valor terico) y
FEV1/FVC 84%. Se realiza una TC torcico que
se muestra en la imagen. Qu diagnstico es el
ms probable en este caso?
1.
2.
3.
4.
5.

8.

Sarcoidosis.
Fibrosis pulmonar idioptica.
Silicosis.
Histiocitosis X.
Neumonitis intersticial inespecfica.

1. Baciloscopia y cultivo de esputo.


2. Determinacin de anticuerpos anti-Hu en
suero.
3. Tomografa computarizada (cerebro, trax y
abdomen).
4. Gammagrafa pulmonar de ventilacinperfusin.
5. Fibrobroncoscopia.

Pregunta vinculada a la imagen n4


En relacin con la enfermedad del caso anterior,
cul de las siguientes afirmaciones es la correcta?
10.
1. Est indicada la realizacin de una gammagrafia pulmonar con Galio 67 para establecer
el diagnstico definitivo.
2. La difusin pulmonar (DLCO) habitualmente
estar disminuda.
3. Una biopsia pulmonar mostrar un patrn
histolgico de neumonitis intersticial descamativa.
4. La causa ms frecuente de muerte es la infeccin respiratoria.
5. Los esteroides suelen modificar la evolucin
de la enfermedad.

9.

Pregunta vinculada a la imagen n5


De las alternativas teraputicas iniciales, cul
es, probablemente, la ms indicada en este caso?
1. Prednisona iv (1 mg/Kg/da) y ciclofosfamida en bolus (500 mg/m2/tres semanas).
2. Ciruga.
3. Isoniazida (300 mg/da), rifampicina (600
mg/da), pirazinamida (25 mg/kg/da) y etambutol (25 mg/K/da).
4. Carboplatino con etopsido.
5. Inmunoglobulinas iv y plasmafresis.

Pregunta vinculada a la imagen n5

11.

Varn de 69 aos, agricultor jubilado; bebedor


importante y fumador de ms de 20 cigarrillos/da desde su juventud hasta hace dos aos en
que abandon el consumo tras ser diagnosticado
de EPOC grave (grado III de GOLD). Es hipertenso, diabtico y presenta dislipemia por lo que
recibe tratamiento con IECAs, estatinas y met-

Pregunta vinculada a la imagen n6


Paciente de 70 aos con antecedentes de carcinoma de colon intervenido quirrgicamente y
sometido a quimioterapia 2 meses antes que
acude a consulta refiriendo lumbalgia progresiva
que interfiere el descanso. Exploracin fsica:
limitacin marcada de movilidad lumbar y T
37,4 C. Presenta la siguiente radiografa repro-

-3-

2. Inmovilizacin durante 3 semanas, y fisioterapia.


3. Artroplastia parcial de hombro.
4. Enclavado intramedular.
5. Artroplastia total de hombro.

ducida en la imagen. Cal de las siguientes es la


prueba diagnstica ms correcta a llevar a cabo
en esta situacin?
1. Gammagrafa sea con MDP-Tec99m y
Galio.
2. Tomografa por emisin de positrones (PET).
3. Densitrometa sea.
4. Biopsia y examen patolgico y microbiolgico.
5. Determinar marcadores tumorales de neoplasia de colon.

12.

15.

Una paciente de 47 aos, carnicera, consulta por


una clnica dos-tres aos de evolucin de debilidad en las manos. Simultneamente, experimenta una sensacin de "entumecimiento" que ha
atribuido al contacto con la carne fra procedente del frigorfico, pero que ltimamente tambin
experimenta en circunstancias no relacionadas
con su trabajo (abrir botellas, por ejemplo). El
interrogatorio dirigido pone de manifiesto cuatro cadas "casuales" en el ltimo ao. La exploracin fsica muestra una leve ptosis palpebral y
debilidad en el giro de la cabeza, con atrofia de
los esternocleidomastoideos. Tambin presenta
una discreta paresia a la flexin dorsal de ambos
pis. Los reflejos osteotendinosos estan presentes
y simtricos. Al solicitarle que abra bruscamente
las manos tras mantenerlas fuertemente cerradas durante unos segundos, lo hace de una forma lenta y dificultosa, tal como puede observarse en la figura. Cal de las siguientes entidades
es la que con ms probabilidad presenta la paciente?

Pregunta vinculada a la imagen n6


En el caso previamente expuesto, cal es la
actitud teraputica ms adecuada?
1. Practicar una artrodesis lumbar con hueso
autlogo.
2. Antibioterapia parenteral.
3. Iniciar calcio ms vitamina D y bifosfonatos.
4. Radioterapia.
5. Reposo y AINEs y revalorar al paciente en
un mes.

13.

Pregunta vinculada a la imagen n7


Hombre de 49 aos con antecedentes personales
de diabetes mellitus tipo 1 acude a Urgencias
tras haber sufrido una cada de unos 2 metros de
altura hace unas 4 horas. Presenta dolor e impotencia funcional del hombro derecho. A la exploracin se aprecia una actitud del hombro en
rotacin interna y deformidad en charretera. La
exploracin neurovascular del miembro es normal. A la vista de la radiologa simple que se
proporciona, cul es el diagnstico?

1.
2.
3.
4.
5.

16.

Distrofia muscular de Duchenne.


Distrofia miotnica.
Miotona congnita.
Polimiositis.
Esclerosis lateral amiotrfica.

Pregunta vinculada a la imagen n8


Cal de las siguientes puebas diagnsticas es la
ms adecuada para establecer el diagnstico?

1. Fractura aislada de troquter mnimamente


desplazada.
2. Luxacin glenohumeral posterior.
3. Fractura-luxacin de extremo proximal de
hmero.
4. Luxacin anterior glenohumeral asociada a
fractura de troquter.
5. Fractura-luxacin de extremo proximal de
hmero con necrosis.

14.

Pregunta vinculada a la imagen n8

1. Estudio gentico de expansin de tripletes de


ADN.
2. Imagen por resonancia magntica de los
diversos grupos musculares.
3. Potenciales evocados somatosensoriales.
4. Estudio bioqumico completo que incluya
cretincinasas y aldolasas.
5. Biopsia muscular con estudios inmunohistoqumicos.

Pregunta vinculada a la imagen n7


Teniendo en cuenta su diagnstico anterior cul
de los siguientes es el tratamiento adecuado para
esta lesin?

17.

Pregunta vinculada a la imagen n9


Hombre de 43 aos que consulta por un cuadro
de debilidad de miembro inferior izquierdo al
caminar y urgencia urinaria. Empeoramiento en
los dos ltimos aos. En la exploracin se objetiva una paraparesia espstica asimtrica de pre-

1. Reduccin cerrada y fijacin del troquter si


permanece desplazado.
-4-

dominio izquierdo y una alteracin de la sensibilidad vibratoria de ambos miembros inferiores.


Se realiza una resonancia magntica cerebral y
cervical. Cul es el diagnstico ms probable
del paciente?
1.
2.
3.
4.
5.

18.

ANCAS postivos. La imagen procede de la biopsia renal practicada. El estudio mediante inmunofluorescencia no demuestra anticuerpos antimembrana basal ni complejos inmunes. Cul
sera su primer diagnstico?

Mielopata vascular.
Esclerosis mltiple primaria progresiva.
Mielopata por dficit de vitamina B12.
Mielopata en el contexto de una colagenosis.
Neuromielitis ptica (sndrome de Devic).

1. Glomerulonefritis proliferativa difusa aguda


postinfecciosa.
2. Glomerulonefritis rpidamente progresiva de
tipo pauciinmune.
3. Glomerulonefritis
membranoproliferativa
tipo II.
4. Sindrome de Goodpasture.
5. Enfermedad de cambios mnimos.

Pregunta vinculada a la imagen n9


Despus de llegar al diagnstico definitivo del
paciente, seale cul es el tratamiento ms apropiado en la actualidad para el enfemo:

22.

Paciente de 42 aos que debuta con hipertencin


arterial, edemas, hematuria y proteinuria moderada. No se observa hemorragia pulmonar. El
estudio mediante inmunofluorescencia demuestra anticuerpos antimembrana basal de confomacn lineal y fibringeno en el espacio capsular
de Bowman. La imagen procede de la biopsia
que se practica. Cul es su primer diagnstico?

1.
2.
3.
4.

Inmunoglobulinas intravenosas.
Vitamina B12 parenteral.
Interfern beta.
Tratamiento sintomtico de la espasticidad y
de la vejiga neurgena.
5. Corticoides intravenosos.

19.

1. Sndrome de Goodpasture.
2. Glomerulonefritis rpidamente progresiva de
tipo pauciinmune.
3. Nefropata lpica.
4. Glomerulonefritis con semilunas asociada a
depsitos inmunes.
5. Glomerulonefritis membranosa evolucionada.

Pregunta vinculada a la imagen n10


Una paciente de 40 aos solicita un tratamiento
de fertilidad por no poder quedarse embarazada. En la anamesis refiere cefaleas frecuentes y
alteraciones visuales progresivas. En el estudio
se solicita entre otras prueba una radiografa de
crneo. Que alteracin observa en la imagen
radiolgica?
1.
2.
3.
4.
5.

20.

23.

Hipoplasia de la base del craneo.


Aumento de tamao de la silla turca.
Lesin osteoltica.
Lesin proliferativa osteoblstica.
Imagen en "sal y pimienta".

Pregunta vinculada a la imagen n12


La radiografa que se adjunta corresponde a un
paciente de 24 aos de edad, adicto a drogas por
va parenteral, que presenta un cuadro clnico de
tres semanas de evolucin de disnea, fiebre, tos y
dolor torcico, sugerente de un proceso infeccioso. Cul sera, entre los siguientes, el primer
diagnstico a considerar?

Pregunta vinculada a la imagen n10


Qu determinacin solicitara para confirmar
el diagnstico?

1. Aspergilosis pulmonar angioinvasora.


2. Tuberculosis pulmonar bilateral.
3. Embolismo sptico pulmonar por Stafiloccocus aureus.
4. Infeccin pulmonar por Nocardia.
5. Neumona bilateral por Pseudomonas.

1. Inmunoelectroforesis srica.
2. Niveles sricos de paratohormona.
3. Niveles sricos de fosfatasa alcalina total y
de la fraccin sea.
4. Niveles sricos de prolactina.
5. Estudio cromosmico.
24.
21.

Pregunta vinculada a la imagen n11

Pregunta vinculada a la imagen n12


Qu sustrato clnico o inmunolgico tiene, entre
los siguientes, una relacin etiopatognica ms
estrecha con el proceso actual de este paciente?

Pregunta vinculada a la imagen n11


Paciente que debuta con hipertensin arterial,
edemas, hematuria, proteinuria moderada y
-5-

1. Alteracin profunda de la inmunidad humoral.


2. Endocarditis tricuspdea.
3. Hepatitis por virus C.
4. Sndrome de inmunodeficiencia adquirida.
5. Leucopenia grave.

25.

que verosmilmente condiciona el cuadro?


1.
2.
3.
4.
5.

Pregunta vinculada a la imagen n13


Paciente de 35 aos con diagnstico de espondiloartropata HLA B-27 + tratada en Reumatologa que acude a urgencias oftalmolgicas por
disminucin de agudeza visual subaguda en ojo
derecho de 24 horas de evolucin con ojo rojo y
dolor. A la exploracin la agudeza visual es de
0,3 en dicho ojo. La imagen asociada a este texto
es lo que observamos en la lmpara de hendidura. La presin intraocular es de 10 mmHg y el
fondo de ojo se distingue con dificultad. La exploracin del ojo izquierdo es normal. Cual es
el diagnstico ms probable en ojo derecho?

28.

1.
2.
3.
4.
5.

Queratoconjuntivitis infecciosa.
Uvetis anterior aguda.
Distrofia corneal.
Queratopata bullosa.
Glaucoma agudo.

Basndonos en el caso de la pregunta anterior se


debe instaurar un tratamiento lo antes posible
para evitar complicaciones irreversibles. Cul
es el tratamiento ms efectivo?

27.

Penicilina G sdica IV.


Doxiciclina.
Cotrimoxazol.
Gentamicina.
Ceftriaxona.

Pregunta vinculada a la imagen n15


Hombre ADVP de 32 aos de edad que acude a
urgencias por fractura de peron. La exploracin fsica general rutinaria muestra hepatomegalia. La analtica muestra una ligera disminucin de las protenas totales y la albmina con un
incremento en los valores de ALT y AST. Los
anticuerpos sricos anti-VHC son positivos as
como los anti-VHBs. Tras el tratamiento de su
fractura se le realiza una biopsia heptica percutnea que se muestra en la imagen. Cul cree
que es el diagnstico ms probable?

Pregunta vinculada a la imagen n13

1.
2.
3.
4.
5.

Pregunta vinculada a la imagen n14


En estas circunstancias el tratamiento ms adecuado para esta enferma es:

29.
1.
2.
3.
4.
5.

26.

Treponema pallidum.
Borrelia burgdorferi.
Bartonella henselae.
Rickettsia conorii.
Coxiella burnetii.

1.
2.
3.
4.
5.

Antinflamatorios no esteroideos sistmicos.


Antibioticos sistmicos de amplio espectro.
Antivirisicos tpicos y a veces sistmicos.
Corticoides tpicos y midriticos.
Lgrimas artificiales a demanda.

30.

Pregunta vinculada a la imagen n14

Cirrosis en el contexto de una hepatitis C.


Necrosis heptica submasiva por txicos.
Hepatitis crnica B sin actividad.
Hepatitis crnica activa C.
Hepatitis por VIH sin actividad en el momento actual.

Pregunta vinculada a la imagen n15


Cul de las siguientes respuestas es la correcta
en relacin con las posibilidades evolutivas en
este caso?

Mujer de 46 aos que vive en un lugar de la


Mancha, acude a urgencias en el mes de agosto.
Refiere que tres das antes, de manera brusca,
comienza con fiebre de 40 acompaada de cefalea intensa y hoy nota la aparicin de manera
diseminada de las lesiones que se exponen en la
porcin inferior de la fotografia, que tambin
afectan palmas y plantas. A la exploracin se
observa, adems de estas lesiones, en cara posterior de muslo derecho otra lesin mostrada en la
porcin superior de la fotografa. El examen
fsico no revel otras anomalas. Los anlisis
mostraron una hemoglobina de 14,1 gr/dL, leucocitos 4.300 mm3, con discreta linfopenia. GOT
68 U/L. GPT 47 U/L. Las radiografias de trax
fueron normales. Cal es el agente etiolgico

1. La esperanza de vida de este paciente, dejado


a su libre evolucin, puede cifrarse en 2-3
aos.
2. Sin tratamiento alguno, el riesgo de desarrollar un hepatocarcinoma a largo plazo es
significativo.
3. El riesgo de padecer hepatitis fulminante es,
por lo menos, un 15 %, por lo que hay indicacin para inclusin inmediata en programa
de trasplante.

-6-

4. El paciente padece un tipo de enfermedad


limitada exclusivamente al hgado, por lo que
no es de esperar patologa extraheptica en el
futuro.
5. Los tratamientos actuales de esta enfermedad
no permiten esperar mejoras duraderas.

31.

bial, bilaterales asociada a un incremento del


nmero de deposiciones y dolor abdominal. La
actitud ms adecuada en este caso es:

1. Recomendar reposo relativo y paos calientes en ambas piernas y aadir tratamiento


antidepresivo.
2. Biopsiar zonas de piel alejados de las reas
lesionadas y pautar analgsicos opioides de
entrada.
3. Sospechar la existencia de una lesin tumoral
maligna intestinal como desencadenante del
proceso cutneo.
4. Sospechar isquemia bilateral de extremidades
inferiores de origen medicamentoso.
5. Ajuste del tratamiento de la enfermedad
intestinal.

La enfermedad por reflujo gastroesofgico puede producir sndromes extraesofgicos, algunos


de ellos estn establecidos o confirmados, y
otros, slo estn propuestos. Cul de ellos no es
un sndrome extraesofgico establecido?

1.
2.
3.
4.
5.

Sinusitis.
Erosin dental por reflujo.
Asma.
Laringitis.
Tos crnica.
35.

32.

En relacin al Helicobacter pylorii, cul de las


siguientes afirmaciones es falsa?

1. La transmisin es por va oral-oral o fecaloral.


2. Se asocia casi siempre a gastritis crnica.
3. Es un microorganismo microaerfilo gramnegativo.
4. Est diseado para vivir en el medio cido
del estomago.
5. El test de ureasa para el diagnstico del H.
pylori presenta una sensibilidad y especificidad inferior al 50%.

33.

1.
2.
3.
4.
5.

36.

Cul de los siguientes datos clnicos, registrados


durante la realizacin de la historia clnica de un
paciente con diarrea de ms de 4 semanas de
evolucin, puede ser encontrado en personas sin
patologa orgnica:

Hemicolectoma derecha.
Hemicolectoma derecha ampliada.
Colectoma subtotal.
Colectoma total con anastomosis ileorrectal.
Colectoma total con anastomosis ileoanal.

Una mujer de 55 aos, postmenopusica, consulta por astenia y disnea de esfuerzo. En la anamnesis refera ligera epigastralgia y pirosis ocasional. No metrorragias. En la analtica: Hb 6
gr/dL, VCM 69 fl, sideremia 13 microgramos/dL, ferritina 4 ngr/mL. Endoscopia digestiva alta: pequea hernia hiatal por deslizamiento
sin signos de esofagitis. Cal es la actitud ms
correcta con esta enferma?.

1. Administrar hierro oral y ver evolucin de la


anemia.
2. Tratar con inhibidores de la bomba de protones y evaluar a los tres meses.
3. Recomendar una colonoscopia completa.
4. Realizar un estudio con cpsula endoscpica.
5. Solicitar un evaluacin ginecolgica.

1. Prdida de peso con o sin rectorragia.


2. Persistencia de la diarrea durante la noche.
3. Inicio de los sntomas despus de los 50
aos.
4. Presencia durante la exploracin de dermatitis y/o artritis.
5. Expulsin de moco en ms del 25% de las
defecaciones.
37.
34.

Una mujer de 35 aos es diagnosticada de cncer


de colon localizado en ngulo heptico. Presenta
antecedentes familiares de cncer de colon en
madre, una ta a la edad de 45 aos y un abuelo.
La intervencin quirrgica ms aceptada es:

Un muchacho de 17 aos de edad con enfermedad de Crohn con afectacin colnica de 2 aos
de evolucin, en tratamiento de mantenimiento
con azatioprina, consulta por la aparicin desde
hace 5 das de ndulos subcutneos rojo violceos, calientes, dolorosos, de localizacn preti-7-

Hombre de 52 aos que consulta para una segunda opinin sobre la necesidad de realizarse
biopsia heptica para estudio de hipertransaminasemia detectada hace dos aos en anlisis
rutinarios de empresa. Entre sus antecedentes
familiares destaca el fallecimiento de su padre
por cirrosis heptica de etiologa no filiada.
Asintomtico y realizando vida social y laboral

sin limitaciones. Niega consumo de alcohol. En la


exploracin fsica destaca pigmentacin metlica
de piel y mnima hepatomegalia no dolorosa.
Resto de la exploracin fsica normal. ndice de
masa corporal 23. Aporta analtica con los siguientes resultados: bilirrubina, albmina, transaminasas AST y ALT, hemograma y tiempo de
protrombina normales; glucemia 150 mg/dl ;
ferritina srica 950 ng/mL; saturacin de transferrina > 45 %. Estudio de virus hepatotropos
negativo. Ecografa abdominal normal. Le han
realizado estudio gentico del gen HFE siendo
homocigoto para la mutacin C282Y. Cual
sera la recomendacin ms acertada con la informacin disponible?:

1.
2.
3.
4.
5.

Cul de estos criterios no es correcto como predictor de mal pronstico:

1.
2.
3.
4.
5.

40.

41.
38.

Hombre de 64 aos con cirrosis heptica Child A


sin antecedentes de descompensacin. Tras deteccin de una lesin focal heptica en ecografa
de cribado se completa el estudio con una TC
toracoabdominal. En esta exploracin se objetiva la presencia de 4 lesiones hepticas (una de
ellas de hasta 6 cm) con patrn de captacin
tpico de hepatocarcinoma, invasin vascular
tumoral y una metstasis en glndula suprarrenal derecha. No se observa ascitis. El paciente
refiere nicamente astenia, pero no presenta
sndrome general. El tratamiento de eleccin en
este paciente sera:

42.

Litiasis vesicular.
Hipertrigliceridemia.
Alcohol.
Hipercolesterolemia.
Traumatismo abdominal.

La causa ms frecuente de fstula colovesical es:

1.
2.
3.
4.
5.

1. Tratamiento sintomtico y de soporte, ya que


presenta un hepatocarcinoma avanzado con
invasin vascular y metstasis.
2. Quimioembolizacin transarterial (TACE),
ya que este tratamiento aumenta la supervivencia en los pacientes con hepatocarcinoma
avanzado.
3. Quimioterapia sistmica con doxorrubicina iv
ya que presenta enfermedad diseminada pero
mantiene buen estado general.
4. Tratamiento con sorafenib oral, ya que se
trata de un enfermo con buen estado general,
Child A y hepatocarcinoma en estadio avanzado BCLC-C.
5. El tratamiento adecuado es el trasplante
heptico ya que es el nico procedimiento
que elimina completamente el tumor primario
y evita las complicaciones futuras de la cirrosis.

39.

Todas las siguientes estn descritas como causa


de pancreatitis aguda, excepto:

1.
2.
3.
4.
5.

Realizar biopsia heptica.


Realizar resonancia magntica heptica.
Iniciar tratamiento con flebotomas.
Iniciar tratamiento con desferroxamina.
Iniciar tratamiento con vitamina E.

Glucemia > 200 mg/dl.


Leucocitosis > 16.000/mm3.
Edad < 55 aos.
LDH srica > 350 UI/L.
GOT srica > 250 UI/L.

La colitis ulcerosa.
La enfermedad de Crohn.
El cncer de colon.
La diverticulitis de colon.
La ciruga abdominal.

Mujer de 26 aos de edad, que presenta incontinencia fecal despus de un parto prolongado e
instrumental. Se practica ecografa endoanal
aprecindose una seccin del esfnter anal externo de 30 de amplitud. El estudio electrofisiolgico demuestra un inervacin normal. Cal es el
tratamiento indicado?.

1. Tratamiento mdico con normas higienodieteticas.


2. Biofeedback esfinteriano.
3. Esfinteroplastia quirrgica.
4. Reparacin quirrgica del suelo plvico.
5. Esfnter anal artificial.

43.

Sobre las hernias de la regin inguinocrural es


FALSO que:

1. Ante una hernia incarcerada est indicada la


intervencin quirrgica urgente, ya que por
definicin presentan compromiso vascular.
2. Actualmente las tcnicas de reparacin herniaria ms generalizadas son aquellas que
emplean material protsico, como la hernioplastia tipo Liechtenstein.

Los criterios de Ranson sirven como marcadores


pronsticos tempranos en la pancreatitis aguda.
-8-

3. Una hernia es de tipo indirecto si el saco


herniario sale de la cavidad abdominal a
travs del orificio inguinal profundo.
4. Las hernias crurales presentan mayor riesgo
de incarceracin que las inguinales.
5. Las hernias de tipo directo se deben a la
debilidad de la fascia transversalis, y aparecen mediales a los vasos epigstricos.

44.

Soplo funcional.
Estenosis artica.
Insuficiencia artica.
Miocardiopata hipertrfica.
Comunicacin interventricular.

Bloqueo aurculo-ventricular de tercer grado.


Estenosis valvular artica degenerativa.
Insuficiencia valvular mitral.
Miocardiopata hipertrfica.
Miocardiopata dilatada.

Una mujer de 78 aos con antecedentes de hipertensin arterial en tratamiento con amlodipino
ingres hace 3 meses por debut de insuficiencia
cardiaca y fibrilacin auricular. Acude a su consulta para revisin cardiolgica. El informe de
alta muestra el resultado del ecocardiograma
que evidencia disfuncin ventricular izquierda
moderada y el tratamiento al alta: se suspendi
el amlodipino y se inici furosemida y enalapril.
La paciente refiere encontrarse mejor sin cansancio y con menos edema de piernas pero todava no est del todo bien. A la exploracin fsica
comprueba que no existen signos de insuficiencia
cardiaca pero llama la atencin unas cifras de
presin arterial y frecuencia cardiaca de 150/90
mmHg y 120 lpm, respectivamente. El ECG
muestra una fibrilacin auricular a 110-120 lpm.
Qu tratamiento tiene ms probabilidades de
mejorar los sntomas y reducir la mortalidad por
eventos cardiovasculares?

1.
2.
3.
4.
5.

Paciente de 78 aos diagnosticado de miocardiopata dilatada idioptica con disfuncin ventricular izquierda ligera (fraccin de eyeccin
48%) y fibrilacin auricular crnica, cul de los
siguientes frmacos debera ser evitado en su
tratamiento?

1.
2.
3.
4.
5.

47.

48.

Un paciente de 81 aos consulta por un episodio


de sncope. Refiere disnea de esfuerzo desde hace
un ao. En la auscultacin cardiaca destaca un
soplo sistlico 3/6 en borde esternal izquierdo
que irradia a cartidas y punta. Qu patologa
le parece ms probable?.

1.
2.
3.
4.
5.

46.

1. Revascularizacin coronaria quirrgica.


2. Reemplazo valvular mitral.
3. Implante de un baln artico de contrapulsacin.
4. Implante de un dispositivo de asistencia
ventricular.
5. Implante de un sistema de resincronizacin
cardiaca.

Un hombre de 26 aos sin antecedentes de inters, asintomtico, presenta en una revisin un


soplo mesosistlico en foco artico y borde paraesternal izquierdo que aumenta con la maniobra de Valsalva. La TA y los pulsos perifricos
son normales y el ECG muestra signos de marcado crecimiento ventricular izquierdo y ondas
T negativas muy profundas en V3, V4, V5 y V6.
Cul es el diagnstico ms probable?

1.
2.
3.
4.
5.

45.

fraccin de eyeccin del ventrculo izquierdo de


26%, insuficiencia mitral grado II-III/IV, sometido a tratamiento con furosemida, espironolactona, enalapril y bisoprolol y que se mantiene en
clase funcional III de la NYHA?

49.

Cul de entre los los siguientes hallazgos debe


excluirse como complicacin del infarto agudo
de miocardio?

1.
2.
3.
4.
5.

Digoxina.
Carvedilol.
Acenocumarol.
Enalapril.
Ibuprofeno.

50.

Qu intervencin teraputica considerara en


un paciente de 67 aos con miocardiopata dilatada idioptica, bloqueo de rama izquierda,
-9-

Digoxina y cido acetilsaliclico.


Atenolol y cido acetilsaliclico.
Verapamilo y acenocumarol.
Carvedilol y acenocumarol.
Carvedilol y clopidogrel.

Endocarditis.
Insuficiencia cardiaca.
Arritmias y bloqueos.
Pericarditis.
Insuficiencia mitral aguda.

Hombre de 72 aos, exfumador, hipertenso y


diabtico. Antecedentes de ictus con paresia
residual en el brazo izquierdo hace 4 meses.
Acude al Servicio de Urgencias de un hospital

comarcal con dolor precordial intenso de 2 horas


de evolucin; en la exploracin fsica presenta
piel fra y sudorosa, tensin arterial de 80
mmHg; electrocardiograma con elevacin marcada del segmento ST en las derivaciones V1-V6,
I y aVL. Cul de los siguientes tratamientos de
reperfusin coronaria es el ms adecuado?.

patologa debe sospechar?

1.
2.
3.
4.
5.

1. Fibrinolisis sistmica de forma inmediata en


el hospital comarcal.
2. Traslado inmediato a un centro terciario para
tratamiento con fibrinolisis.
3. Traslado inmediato a un centro terciario para
procedimiento de revascularizacin percutnea.
4. Estabilizacin hemodinmica y procedimiento de revascularizacin diferido.
5. Tratamiento inmediato con fondaparinux y
abciximab.

51.

53.

Las arritmias ventriculares.


La insuficiencia cardaca.
Los bloqueos AV de 2 y 3 grados.
La hipercolesterolemia.
La valvulopata artica degenerativa.

55.

Hombre de 75 aos, hipertenso y dislipmico con


antecedentes de insuficiencia cardiaca por disfuncin sistlica de ventrculo izquierdo
(FE<30%). Acude a nuestra consulta por empeoramiento de su disnea habitual en las ltimas
semanas. Su tensin arterial sistlica es de
160/95 mmHg con frecuencia cardiaca de 65
lpm. En la exploracin fsica no se auscultan
crepitantes y su presin venosa yugular es normal. Aporta una determinacin de creatinina
que es de 3.7 mg/dl con iones dentro de la normalidad. Cal de los siguientes frmacos es el
ms adecuado para mejorar el pronstico de su
insuficiencia cardiaca?

1.
2.
3.
4.
5.

Un hombre de 60 aos de edad refiere dolor


precordial tras los medianos esfuerzos. La coronariografa muestra estenosis significativa en los
segmentos proximales de los tres vasos principales con buen lecho distal. La funcin ventricular
izquierda est deprimida (< 30%). Cul es la
mejor opcin teraputica?

1.
2.
3.
4.
5.

Los inhibidores de la convertasa son frmacos de


primera lnea para el tratamiento de:

1.
2.
3.
4.
5.

52.

54.

Cardiopata isqumica.
Miocardiopata dilatada.
Estenosis valvular artica severa.
Pericarditis constrictiva.
Derrame pericrdico con taponamiento cardaco.

Revascularizacin percutnea.
Tratamiento mdico.
Ciruga de revascularizacin miocrdica.
Implantar baln de contrapulsacin.
Trasplante cardiaco.

Hombre de 64 aos que consulta por dolor en la


pantorrilla derecha cuando sube cuestas y escaleras y cuando anda en llano durante un tiempo
prolongado. Refiere que el dolor se hace tan
intenso con el esfuerzo que le obliga a pararse, lo
que hace que mejore el dolor en cuestin de minutos. Como antecedentes personales, destaca
que el paciente es fumador de 20 cigarrillos al
da, diabtico en tratamiento con metformina e
hipertenso. A la exploracin, se objetiva una
buena perfusin perifrica del pie derecho, pero
presenta un pulso pedio disminudo. Qu prueba complementaria, entre las siguientes, debe
solicitar inicialmente para el diagnstico y valoracin de la gravedad de la enfermedad de este
paciente?

1. Arteriografa de extremidades inferiores.


2. ndice tobillo-brazo.
3. Tomografa axial computerizada con reconstruccin vascular de extremidades inferiores.
4. Prueba de esfuerzo en cinta.
5. Resonancia magntica.

Enalapril.
Losartan.
Espironalactona.
Bisoprolol.
Eplerenona.

56.

Un paciente con antecedentes de cuadro febril y


dolor torcico acude al hospital con disnea y
taquipnea. En la exploracin fsica las cifras de
presin arterial estn bajas, la presin venosa
yugular est elevada con un seno "X" descendente profundo y tiene pulso paradjico. Qu
- 10 -

Una mujer de 58 aos acude para una visita de


seguimiento por diabetes mellitus e hipertensin.
Se siente bien pero afirma que ha dejado de
tomar el verapamilo por estreimiento. Presenta
intolerancia a los IECAs por tos. En la exploracin, la presin arterial es de 156/92 mm Hg. En
la analtica incluyen una creatinina de 1.6
mg/dL, excrecin de protenas en orina de 24
horas de 1.5 g/da y un aclaramiento de la crea-

tinina de 45ml/min. Sobre esta base cal es el


tratamiento ms eficaz para enlentecer la progresin de la nefropata diabtica tipo 2 de la
paciente?

gencias por disnea. En la gasometra realizada


respirando aire ambiente se observa un pH:
7.45, PaO2 56 mmHg, PaCO2 30 mmHg, HCO3
26 mmol/L. Qu alteracin gasomtrica presenta la paciente?

1. Inhibidor de la enzima convertidora de la


angiotensina.
2. Bloqueador del receptor de la angiotensina.
3. Antagonista del calcio.
4. Alfa-bloqueante.
5. Beta-bloqueante.

57.

1. Insuficiencia respiratoria hipoxmica con


alcalosis respiratoria compensada.
2. Alcalosis metablica aguda con insuficiencia
respiratoria hipoxmica e hipocpnica.
3. Insuficiencia respiratoria normocpnica crnica, sin alteracin del equilibrio cido-base.
4. Insuficiencia respiratoria hipoxmica crnica
con acidosis metablica compensada.
5. Insuficiencia respiratoria hipoxmica, con
alcalosis mixta compensada.

El choque cardigeno se caracteriza por un crculo vicioso en que la depresin de la contractilidad del miocardio (por lo comn por isquemia)
hace que disminuya el gasto cardiaco y la presin arterial lo que a su vez origina deficiente
perfusin miocrdica. De entre las siguientes
medidas cite la que no ha demostrado mejorar la
supervivencia de los pacientes:

60.

1. Oxigenacin y respiracin artificial para


combatir la congestin pulmonar retrgrada.
2. Medicacin antiinflamatoria para combatir el
sndrome de respuesta inflamatoria sistmica.
3. Contrapulsacin intraartica para mejorar el
gasto cardaco y la presin arterial media.
4. Cuando la causa es isqumica, reperfusin
miocrdica urgente mediante angioplastia
percutnea, frmacos fibrinolticos o ciruga
de revascularizacin.
5. Ciruga correctora urgente cuando la causa
del shock sea la insuficiencia mitral, la rotura
del tabique o la de la pared ventricular.

58.

1. Traslado a la UVI.
2. Aerosoles con beta agonistas en el rea de
Urgencias para escalar progresivamente el
tratamiento mdico segn respuesta.
3. Ingreso inmediato en planta porque es poco
probable que se recupere antes de 4-5 das.
4. Poner 40 mg. iv de metil-prednisolona y
mandar a domicilio con inhalaciones de salbutamol a demanda.
5. Antes de tomar decisiones teraputicas, realizar radiografa de trax y hemograma.

Cal de estas alteraciones funcionales resulta


sugestiva de debilidad diafragmtica?

1. Alteracin de los flujos mesoespiratorios


forzados sin afectacin del volumen espiratorio forzado en un segundo (FEV1) y de la
capacidad vital forzada (FVC).
2. Disminucin importante del FEV1/FVC.
3. Disminucin de la difusin para el monxido
de carbono (DLCO) relacionada con el volumen alveolar.
4. Disminucin de la FVC con empeoramiento
del resultado obtenido en decbito supino
respecto a la sedestacin.
5. Mejora de la alteracin de la capacidad pulmonar total (TLC) tras ejercicio moderado.

59.

Una mujer de 23 aos de edad, con antecedentes


de asma extrnseca desde nia, con varios ingresos hospitalarios en los ltimos aos por este
motivo, acudi hace dos semanas de una consulta rutinaria en el Servicio de Alergia, donde una
espirometra fue normal. Acude al Servicio de
Urgencias presentando un nuevo episodio de
disnea, ruido al respirar y tos improductiva,
instaurado en el plazo de las 48 horas previas. A
su llegada la TA es de 130/70 mm Hg. Pulso 120
lpm regular y rtmico, saturacin capilar de
oxgeno 93 %, 22 respiraciones por minuto y, en
la auscultacin pulmonar, espiracin alargada y
sibilancias generalizadas. Se realiza una gasometra en sangre arterial, cuyo resultado nos dan a
conocer enseguida: pH 7,40, PO2 64 mm Hg,
PCO2 44 mm Hg. Bicarbonato 23 mEq/l. Cul,
entre las siguientes, es la decisin ms correcta?

61.

Una mujer de 67 aos con antecedentes de insuficiencia cardaca congestiva (que lleva tratamiento con ramipril y furosemida) y diabetes
tipo 2 (en tratamiento con insulina) acude a ur- 11 -

Un hombre de 65 aos, oficinista jubilado y fumador de 1 paquete diario de cigarrillos, acude a


consulta por un cuadro de tos persistente, generalmente seca y disnea progresiva de 2 aos de
evolucin que en la actualidad es de grado 2. El
paciente niega otros sntomas. La exploracin
fsica no muestra datos relevantes. La actitud a
seguir sera:

1. Iniciar tratamiento con broncodilatadores


inhalados.
2. Realizar radiografa de trax y espirometra
con prueba broncodilatadora.
3. Pautar corticoides orales.
4. Realizar TC torcica.
5. Realizar gasometra arterial basal.

62.

U/dL (cociente LDH pleura/suero 7.01), colesterol 97 mg/dL, triglicridos 67 mg/dL, ADA 116
U/L y tincin de gram sin deteccin de grmenes.
Entre los indicados, cul es su primera sospecha diagnstica?

1.
2.
3.
4.
5.

En un paciente con asma bronquial que presenta


sntomas diurnos diarios, sntomas nocturnos
ms de una noche por semana y que muestra en
su espirometra un volumen espiratorio forzado
en el primer segundo (FEV1) del 70% del valor
predicho. Cal es el tratamiento de mantenimiento ms apropiado?

65.

1. Corticoides inhalados a dosis bajas y agonistas B2-adrenrgicos inhalados de accin


corta.
2. Corticoides inhalados a dosis bajas y agonistas B2-adrenrgicos inhalados de accin
prolongada.
3. Corticoides inhalados a dosis elevadas y
antogonistas de los leucotrienos.
4. Antagonistas de los leucotrienos y agonistas
B2-Adrenrgicos inhalados de accin prolongada.
5. Antagonistas de los leucotrienos y teofilinas.

63.

64.

Un hombre de 37 aos sin antecedentes de inters acude a Urgencias por haber sufrido un sncope mientras caminaba. Haba comenzado dos
das antes con disnea de esfuerzo. Exploracin:
peso 75 kg, TA 75/50 mmHg. Saturacin de oxgeno 89%. Auscultacin pulmonar normal. Auscultacin cardiaca: tonos rtmicos taquicrdicos
a 130 lpm. Abdomen normal. Extremidades: sin
alteraciones. Hemograma: normal. Troponina:
1.2 ng/mL (normal: < 0.04). ECG: Taquicardia
sinusal a 130 lpm. T invertida en V1-V4. AngioTC torcica: defecto de replecin en ambas
arterias pulmonares principales. Una hora despus de su llegada a Urgencias persiste TA 70/55
mmHg. Cul sera el tratamiento inicial ms
adecuado?

1. Heparina no fraccionada, 10000 U iv ante la


sospecha clnica. Fibrinolisis con activador
tisular del plasmingeno (tPA), 100 mg iv,
una vez confirmado el diagnstico.
2. Enoxaparina: 80 mg sc cada 12 horas, comenzando ante la sospecha diagnstica
3. Enoxaparina: 80 mg sc cada 12 horas, comenzando tras la confirmacin diagnstica
4. Fondaparinux, 7.5 mg sc diarios.
5. Tromboendarterectoma pulmonar de urgencia.

La rehabilitacin pulmonar produce todas excepto una de las siguientes acciones en los pacientes con EPOC.

1.
2.
3.
4.
5.

Derrame paraneumnico complicado.


Tromboembolismo pulmonar.
Insuficiencia cardiaca congestiva.
Tuberculosis pleural.
Quilotrax.

Mejora la calidad de vida.


Mejora la disnea.
Mejora la capacidad del ejercicio.
Disminuye la frecuencia de hospitalizacin.
Disminuye la mortalidad.

Un hombre de 37 aos acude a urgencias por


dolor de caractersticas pleurticas en costado
derecho y fiebre vespertina de 38,5 en los ltimos cinco das. Presenta una frecuencia respiratoria de 22 r.p.m. y abolicin del murmullo vesicular en la zona afecta, no encontrndose otras
alteraciones en la exploracin. La radiografa de
trax confirma un derrame pleural que ocupa la
mitad inferior del hemitrax derecho, con silueta
cardaca normal. El electrocardiograma, gasometra, hemograma, bioqumica y coagulacin
con dmero D estn en rango normal. La protena C reactiva es de 12.2 mg/dL (lmite normal
<0.5). Se practica una toracocentesis con los
siguientes resultados en el lquido pleural: 4000
leucocitos/mm3 (5% neutrfilos, 95% linfocitos),
ausencia de clulas mesoteliales, glucosa 75
mg/dL, pH 7.35, protenas 5.6 g/dL, LDH 1094

66.

Un paciente ingresado por pancreatitis aguda,


comienza con taquipnea, taquicardia, sudoracin y cianosis progresiva. La PaO2 es de 55
mmHg (cociente PaO2/FiO2 < 200mmHg). La
radiografa de torax muestra infiltrados alveolares bilaterales. La presin de enclavamiento
capilar pulmonar es normal. La oxigenoterapia
no mejora la situacin. Qu diagnstico es el
ms probable?

1.
2.
3.
4.
5.
- 12 -

Neumona nosocomial.
Insuficiencia cardiaca.
Linfangitis carcinomatosa.
Tromboembolismo pulmonar.
Distress respiratorio.

67.

1. La movilidad espinal rara vez est comprometida.


2. No existe sacroiletis radiogrfica.
3. Hay predominio en mujeres.
4. Es excepcional que afecte a la columna cervical.
5. El curso es agudo e invalidante.

Existen varios trminos semiolgicos que tienen


en cuenta el patrn respiratorio de un paciente.
Si se observa en un paciente, tras un perodo de
apnea, que se producen respiraciones que van
aumentando progresivamente de amplitud y
frecuencia y luego disminuyen progresivamente
hasta otro episodio de apnea. A qu patrn
corresponde?
70.
1.
2.
3.
4.
5.

68.

Respiracin de Cheyne-Stokes.
Taquipnea.
Platipnea.
Respiracin de Kussmaul.
Bradipnea.

1. En el tratamiento inicial puede elegirse entre


esteroides o inmunosupresores
2. La principal complicacin es la amaurosis
por afectacin neurtica isqumica
3. La elevacin de la creatinkinasa es tpica
cuando existe polimialgia asociada
4. La biopsia de la arteria temporal tiene que
resultar positiva para iniciar el tratamiento
5. La arteritis de Takayasu es clnicamente
similar pero la histologa es muy diferente

Una paciente de 42 aos de edad refiere dolor de


caractersticas inflamatorias y tumefaccin en
ambas muecas, 2 y 3 metacarpofalngicas e
interfalngicas proximales de forma bilateral y
tobillo izquierdo de 4 meses de evolucin acompaada de rigidez matutina de ms de una hora
de duracin. En la radiografa de manos se objetiva una erosin en la apfisis estiloides del cbito en el carpo derecho. En la analtica destaca
una Hb: 10 g/dL con VSG de 45 mm en 1 hora,
PCR 16 mg/L, factor reumatoide 160 Ul/mL.
Tras 6 meses de tratamiento con indometacina y
metotrexate la paciente persiste con dolor y tumefaccin de ambos campos, rigidez matutina de
30 minutos de duracin y una analtica donde
destaca una VSG 30 mm en 1 hora y una PCR 9
mg/dL. Respecto a la actitud a tomar, cul de las
siguientes es verdadera:

71.

1. Suspender el tratamiento pautado por la falta


de respuesta e iniciar prednisona a dosis altas
para el control de los sntomas exclusivamente.
2. Mantener la actitud teraputica tomada dado
que slo llevamos 6 meses y habra que esperar un mnimo de 9 meses para valorar respuesta teraputica.
3. Si no existe contraindicacin mdica, valorar
asociar al tratamiento un anti-TNF alfa.
4. Iniciar lo antes posible un segundo frmaco
modificador de la enfermedad dado que no se
podra iniciar tratamiento con terapia biolgica slo tras metotrexate.
5. Valorar iniciar tratamiento con terapia antiCD20 asociada al metotrexate.

69.

Cul de las siguientes afirmaciones respecto a


la arteritis temporal o de clulas gigantes es
cierta?

Una paciente de 25 aos con antecedentes de


rash cutneo tras exposicin solar y poliartritis
ocasionales en articulaciones de las manos que
controla con antiinflamatorios no esteroideos
presenta desde hace 15 das, malestar general,
sensacin progresiva de debilidad generalizada y
palidez. En la analtica destaca una Hemoglobina de 7 g/dL, VCM de 108 mm/h, 150000 plaquetas/mm3, 3000 leucocitos/mm3, LDH elevada, haptoglobina indetectable. En el caso de
elegir una sola prueba diagnstica indique cul
de las siguientes determinaciones debe realizarse
en primer lugar:

1.
2.
3.
4.
5.

72.

Test de Coombs directo.


Anticuerpos antinucleares (ANA).
Vitamina B12.
Ferritina.
cido flico.

Una mujer de 75 aos consulta por lesiones violceas en manos y cuello junto a debilidad muscular progresiva de 3 meses de evolucin. Qu
pruebas diagnsticas, entre las que se indica,
puede ser de utilidad para el diagnstico?

1.
2.
3.
4.

Determinacin de aldolasa srica


Electroencefalograma
Biopsia de tejido celular subcutneo
Determinacin de anticuerpos anti msculo
liso
5. Estudio gentico de sus descendientes

La espondilitis anquilosante se diferencia de la


hiperostosis esqueltica difusa idioptica o enfermedad de Forestier-Rots en que en esta ltima:

- 13 -

73.

vibratoria estn conservadas. No presenta atrofia muscular ni dficit de fuerza. Los reflejos
musculares son normales y simtricos. No se
observa dismetra, disdiadococinesia ni temblor
intencional. El resto de la exploracin neurolgica es rigurosamente normal. Indique cul es el
diagnstico ms probable en este caso:

Un paciente hiperuricmico que habitualmente


toma 100 mg de alopurinol al da acude a urgencias con dolor agudo y signos inflamatorios en
rodilla derecha. Se realiza artrocentesis y en el
microscopio de luz polarizada se observan cristales intracelulares con birrefrignecia negativa.
Qu actitud teraputica entre las que a continuacin se indica, es la ms adecuada en este
caso?

1. Neuropata perifrica sensitiva simtrica


distal de causa diabtica.
2. Lesin compresiva medular cervical.
3. Sndrome del tunel del carpo bilateral.
4. Enfermedad desmielinizante tipo esclerosis
mltiple.
5. Lesin medular central cervical.

1. Suspender alopurinol e iniciar tratamiento


con colchicina.
2. Suspender alopurinol e comenzar con AINE.
3. Aadir un AINE hasta que la cristis remita.
4. Aumentar la dosis de alopurinol a 300
mg/da.
5. Sustituir el alopurinol por un uricosrico.

74.

De los mtodos teraputicos citados a continuacin, cul considera que resulta ms eficaz para
prevenir la fractura de cadera?

1.
2.
3.
4.
5.

75.

77.

1.
2.
3.
4.
5.

Estrgenos.
Calcio y Vitamina D.
Bifosfonatos.
Actividad fsica.
Calcitonina.

78.

Paciente de 75 aos de edad, con importante


gonartrosis con "Genu Varo" que le condiciona
marcada impotencia funcional, limitacin de la
movilidad y dolor contnuo que le exige estar con
medicacin antirreumtica, antiinflamatoria y
analgsicos de forma continuada. Qu tratamiento debe indicarse en este caso?
79.
Lavado artroscpico de la rodilla.
Rehabilitacin de la rodilla afectada.
Artroplastia de la rodilla afectada.
Osteotoma tibial supratuberositaria de abduccin.
5. Sinovectoma de rodilla.

80.

- 14 -

La existencia de tics bucofonatorios.


La existencia de tics motores.
Un nivel intelectual alto.
La existencia de ideas obsesivas.
Los trastornos de conducta.

Un varn de 78 aos consulta por un cuadro de


deterioro cognitivo progresivo de un ao de evolucin con fallos de memoria y de orientacin. Su
familia refiere alucinaciones visuales recurrentes, cadas ocasionales y un enlentecimiento motor llamativo. Cal es el diagnstico ms probable?

1.
2.
3.
4.
5.

Un paciente de 32 aos, diabtico en tratamiento


con insulina y un buen control de sus cifras de
glucemia, acude a su consulta por presentar
hormigueo en ambas manos, con sensacin de
acorchamiento e insensibilidad trmica de instauracin progresiva en el curso de 2 semanas.
No refiere trastornos visuales, dficit de fuerza,
torpeza motora, ni otros sntomas. En la exploracin encuentra una anestesia para el dolor y la
temperatura en ambas manos y porcin distal de
los antebrazos; la sensibilidad posicional y la

Cerebral anterior derecha.


Cerebral media derecha.
Cerebral media izquierda.
Cerebral anterior izquierda.
Cerebral posterior.

Es patognomnico de la enfermedad llamada


Gilles de la Tourette:

1.
2.
3.
4.
5.

1.
2.
3.
4.

76.

Indique qu arteria es la afectada en un paciente


diestro que, por un accidente cerebro-vascular,
sufre trastornos motores, sensitivos y afasia:

Demencia vascular multi-infarto.


Demencia tipo Alzheimer.
Demencia con cuerpos de Lewy difusos.
Demencia fronto-temporal.
Hidrocefalia normotensiva.

Una mujer de 42 aos, casada y con 2 nios en


edad escolar, consulta a su mdico de familia por
un fuerte dolor de cabeza de unos 7 das de evolucin. Aunque ya hace ms de un ao que viene

1. Realizar una ecografa tiroidea antes de iniciar el tratamiento.


2. Determinar la T3 Libre y realizar una resonancia magntica hipofisaria.
3. Tratar con L-Triyodotironina y una estatina.
4. Tratar con L-Tiroxina y determinar anticuerpos antitiroideos.
5. Realizar una citologa tiroidea, previa al
tratamiento.

teniendo episodios similares, en los 2 ltimos


meses se han agravado notablemente. El dolor es
como un peso que comienza en la zona occipital,
se extiende a ambas regiones temporales y apenas se le alivia tomando pastillas de 650 mgr de
paracetamol, por lo que solicita una TC (est
convencida de que algo tiene que tener en la
cabeza). Cul de las siguientes es la causa ms
plausible de su cefalea?

1.
2.
3.
4.
5.

81.

84.

Hematoma subdural crnico.


Fstula cartido-cavernosa.
Hidrocefalia postraumtica.
Fractura de base craneal anterior.
Trombosis del seno venoso sagital.

85.

Cal es la causa ms frecuente de hipoglucemia


en el adulto?

1.
2.
3.
4.
5.

Paciente mujer de 18 aos, con historia de ausencias entre los 6 y 9 aos, crisis tnico-clnicas
generalizadas de reciente comienzo y saltos violentos de miembros superiores al desayunar. La
clnica empeora con salidas nocturnas de fines de
semana. Un EEG muestra descargas de polipuntas agudas a 6 ciclos/segundo. El diagnstico ms
probable es:

86.

1. Gran mal epilptico.


2. Sndrome de Lennox-Gastaut.
3. Epilepsia sintomtica por esclerosis temporal
mesial.
4. Pequeo mal atpico.
5. Epilepsia mioclnica juvenil.

83.

En caso de sospecha clnica de sndrome de Cushing, cal de estas pruebas diagnsticas debe
realizarse inicialmente?

1. Determinacin de cortisol plasmtico basal.


2. Determinacin de corticotropina (ACTH)
plasmtica basal.
3. Determinacin de cortisol libre en orina de
24 horas.
4. Resonancia magntica hipofisaria.
5. Tomografa axial computerizada abdominal.

Si despus de un TCE apareciese exoftalmos


pulstl, soplo audible, intensa quemosis conjuntival, nos indicara la posible existencia de:

1.
2.
3.
4.
5.

82.

La migraa.
Los trastornos vasculares.
La arteritis de la arteria temporal.
La tensional.
La oncolgica.

Hombre de 24 aos que refiere astenia progresiva desde al menos 6 meses, voz ronca, lentitud
del habla, somnolencia e hinchazn de manos,
pies y cara. Exploracin: pulso a 52 latidos por
minuto, cara abotargada y piel seca y plida.
Analtica: Ligera anemia, colesterol 385 mg/dL
(normal <220), creatinina 1,3 mg/dL (normal:
0,5-1,1), proteinuria negativa, TSH 187 UI/mL
(normal 0,35-5,5) y T4 Libre 0,2 ng/dL (normal
0,85-1,86). Qu estrategia le parece ms adecuada?

Ayuno prolongado.
Ejercicio fsico excesivo.
Hipoglucemia reactiva.
Insulinoma.
Iatrognica.

Mujer de 76 aos con obesidad y diabetes mellitus tipo 2 en tratamiento con metformina. Acude
a urgencias porque desde hace tres das presenta
fiebre de 38C, polaquiuria, disuria, intensa sed
y dismunicin progresiva de su nivel de conciencia. En la exploracin fsica se aprecian signos de
deshidratacin, presin arterial 95/54 mmHg y
no hay signos de focalidad neurolgica. En la
analtica destaca leucocitosis, creatinina de 1.8
mg/dL y glucemia 855 mg/dL. Cal es el diagnstico ms probable?

1. Coma hiperglucmico hiperosmolar no cetsico.


2. Insuficiencia cardiaca.
3. Hematoma subdural.
4. Insuficiencia renal de causa obstrutiva.
5. Cetoacidosis diabtica con coma.

87.

- 15 -

Un paciente diabtico de 60 aos consulta por


primera vez respecto al tratamiento de su enfermedad. Qu objetivo a alcanzar de los
siguientes le recomendara en primer lugar?

3. El trastorno gentico afecta tambin a la


sntesis de los triglicridos, provocando en
las fases avanzadas de la enfermedad, un
incremento de los mismos, que suele ser
superior a los 500 mg/dl.
4. Los varones no tratados tienen una probabilidad cercana al 50% de sufrir un evento coronario antes de los 60 aos de edad, y las
mujeres con la enfermedad presentan una
prevalencia de cardiopata isqumica superior
a la de la poblacin general femenina.
5. Las estatinas son eficaces en el tratamiento
de la forma heterocigota de la enfermedad,
pero a menudo necesitan combinarse con
inhibidores de la absorcin de colesterol para
alcanzar los objetivos teraputicos y el control de la enfermedad.

1. Mantener la tensin arterial por debajo de


110/70 mmHg.
2. Abandono del hbito tabquico.
3. Mantener un ndice de masa corporal (IMC)
menor de 21.
4. Realizarse glucemia capilar basal a diario.
5. Evitar las grasas animales en la dieta.

88.

Paciente de 55 aos de edad diagnosticado de


HTA hace 2 meses en revisin de empresa. Analtica: glucosa 129 mg/dl, colesterol 202 mg/dl,
LDLc 160 mg/dl, HDLc 38 mg/dl, triglicridos
171 mg/dl, creatinina 1.1 mg/dl. Consulta por
mal control de la tensin arterial, edemas maleolares y disnea que ocasionalmente es nocturna.
Exploracin fsica: IMC 38 kg/m2, permetro
abdominal 110 cm. TA 157/93, FC 70 lpm, FR 14
rpm. Soplo sistlico con refuerzo del segundo
ruido. Pulsos perifricos con discreta asimetra
en pedio y tibial posterior derechos que son ms
dbiles respecto a extremidad izquierda. Rx de
trax y ECG sin alteraciones. Cul de las siguientes pruebas complementarias es la menos
necesaria para la deteccin de lesiones en rganos diana?

1.
2.
3.
4.
5.

89.

90.

Ecocardiograma transtorcico.
Realizacin de fondo de ojo.
Determinacin de microalbuminuria.
ndice tobillo-brazo.
Determinacin de hemoglobina glicosilada.

Hombre de 34 aos remitido a la consulta por


presentar cifras de colesterol LDL superiores a
250 mg/dl, de forma persistente. El paciente se
encuentra asintomtico pero en la exploracin se
aprecia la presencia de manchas induradas blanquecinas en codos, rodillas y prpados. Su TA es
de 135/85 mmHg. Su padre muri de un infarto
de miocardio a la edad de 48 aos. En la analtica, realizada en ayunas, destaca un colesterol
total de 346 mg/dl, un LDL colesterol de 278
mg/dl, un HDL colesterol de 42 mg/dl, unos triglicridos de 130 mg/dl y una glucosa de 93
mg/dl. Respecto a la enfermedad que padece este
paciente, una de las siguientes afirmaciones es
FALSA. Seale cul:

Una mujer de 55 aos acude a consulta por fatiga y poliuria de dos aos de evolucin. Los anlisis revelan hipercalcemia, aumento de la PTH e
incremento de los marcadores del recambio seo.
El estudio radiogrfico muestra resorcin subperistica y osteoporosis. La gammagrafa con
sestamibi pone de manifiesto un adenoma paratiroideo. Se efecta una paratiroidectoma con
ciruga mnimamente invasiva. En el postoperatorio la enferma desarrolla hipocalcemia intensa
y tetania, siendo la PTH inferior a 5 ng/l. La
enferma responde bien al tratamiento inicial con
calcio intravenoso y posteriormente con calcio y
vitamina D orales. Cal es el diagnstico ms
probable?

1.
2.
3.
4.
5.

91.

1. Se trata de una enfermedad hereditaria autosmica codominante, con una alta penetrancia, por lo que es frecuente que alguno de los
progenitores y hermanos del individuo la
padezcan tambin.
2. El trastorno gentico afecta al gen que codifica el receptor del LDL colesterol y se han
descrito ms de 900 mutaciones.

Hipoparatoroidismo quirrgico permanente.


Transfusin de sangre citratada.
Insuficiencia de vitamina D.
Sndrome del hueso hambriento.
Osteomalacia.

Hombre de 62 aos con antecedentes de hipertensin arterial en tratamiento con captoprilo,


lcera duodenal y urolitiasis por cido rico.
Consulta por crisis de podagra tpica similar a
otras presentadas en los ltimos dos aos. En la
exploracin se obsevan tofos en ambos pabellones auriculares. Los anlisis muestran cido
rico 10.1 mg/dl, creatina 1.5 mg/dL. Indique
cal de las siguientes respuestas es FALSA en
relacin con su posible tratamiento con alopurinol.

1. Su empleo es de segunda eleccin, cuando


han fracasado los uricosricos.
- 16 -

2. Su empleo por tiempo prolongado es prcticamente obligado porque existen tofos.


3. Su introduccin debe demorarse hasta que se
haya resuelto al ataque actual con antiinflamatorios o con colchicina.
4. Su introduccin debe ser gradual hasta una
dosis que consiga una uricemia inferior a 6
mg/dL.
5. En los primeros meses de tratamiento se
aconseja asociar dosis bajar de colchicina
para prevenir nuevos ataques.

92.

1. Aplasia medular e inmunoterapia con timoglobulina y ciclosporina.


2. Esferocitosis hereditaria y esplenectoma.
3. Anemia hemoltica autoinmune asociada a
infeccin respiratoria y corticoides.
4. Anemia perniciosa e inyecciones peridicas
de vitamina B12.
5. Leucemia aguda y quimioterapia.

Cal de los siguientes datos NO es propio del


sndrome hemoltico?

1.
2.
3.
4.
5.

93.

cultando la prueba cruzada. Cal sera su sospecha diagnstica y el tratamiento ms adecuado.

95.

Adenopatas.
Esplonomegalia.
Hiperbilirrubinemia.
Incremento LDH srica.
Reticulocitosis.

1.
2.
3.
4.
5.

Mujer de 35 aos sana que consulta por astenia. En el hemograma presenta anemia microctica e hipocroma (hemoglobina de 7 g/dL, volumen corpuscular medio de 68 fL, hemoglobina
corpuscular media de 24 pg) con una amplitud
de distribucin eritrocitaria elevada (ADE 20%),
reticulocitos disminudos (0.3%, 30.000/L absolutos), contenido de hemoglobina disminudo en
los reticulocitos (17 pg) y discreta trombocitosis
(500.000 plaquetas/uL). Con los datos del hemograma, cul sera su sospecha diagnstica an
sin tener la bioqumica de la paciente?

96.

Azacitidina.
Imatinib.
Bortezomib.
Rituximab.
Zoledronato.

La mutacin del gen JAK-2 constituye uno de


los criterios mayores de diagnstico en una de
estas patologas :

1. Policitemia Vera.
2. Leucemia Mieloide Aguda tipo M4 de la
FAB.
3. Sndrome Mielodisplsico tipo Anemia Refractaria.
4. Leucemia Mieloide Crnica.
5. Leucemia Mielomonoctica Crnica.

1. Rasgo talasmico.
2. Anemia central, arregenerativa, por dficit de
cido flico.
3. Anemia perifrica, regenerativa, hemoltica.
4. Anemia perifrica, regenerativa, por hemorragia activa aguda.
5. Anemia central, arregenerativa, por dficit de
hierro.

94.

Cal de las siguientes drogas es til en el sndrome mielodisplsico?

97.

Mujer de 32 aos con parlisis cerebral del parto que acude a urgencias por cuadro de orinas
oscuras de unos das de evolucin en relacin
con episodio de fiebre alta y tos seca. Al ingreso
se objetiva en el hemograma 16900 leucocitos/mm3 (85%S; 11%L; 4%M), hemoglobina de
6.3 g/dl; VCM 109 fl; 360000 plaquetas/mm3. En
la bioqumica LDH 2408 UI/L; Bilirrunina 6,8
mg/dl (Bilirrubina no conjugada 6,1 mg/dl) GOT
y GPT normales. En el estudio morfolgico de
sangre se observa anisocitosis macroctica con
frecuentes formas esferocitadas y policromatofilia sin blastos. El estudio de anticuerpos irregulares es positivo en forma de panaglutinina, difi-

Una mujer de 43 aos consulta a su mdico de


atencin primaria por cansancio, gingivorragias
y petequias. Se realiza analtica en la que destaca: anemia de 8 g/dl, trombopenia de
4000/microlitro y leucopenia de 1200/microlitro
con neutropenia absoluta. En el estudio de coagulacin se observa alargamiento del APTT
(43), actividad de la protrombina disminuda
(55%), hipofibrinogenemia (98 mg/dl) y presencia de concentracin elevada de dmero-D y monmeros de fibrina. Se remite para estudio
hematolgico urgente, realizndose un aspirado
de mdula sea en el que se observa una infiltracin masiva por elementos inmaduros con ncleo
hendido, y numerosas astillas y bastones de Auer
en el citoplasma. Cul es el diagnstico ms
probable de esta paciente?

1.
2.
3.
4.
- 17 -

Leucemia mieloide crnica.


Leucemia aguda promieloctica.
Leucemia aguda monoblstica.
Leucemia linfoide crnica.

5. Leucemia linfoide aguda.

98.

Entre los linfomas que se citan a continuacin


cul de ellos se presenta con mayor frecuencia
en nuestro medio?

1.
2.
3.
4.
5.

99.

3. El Factor V Leiden en heterocigosis es una


trombofilia de bajo riesgo y no hay necesidad
de ningn tratamiento en el embarazo y puerperio.
4. Se debe aconsejar aspirina a bajas dosis durante todo el embarazo y puerperio.
5. Debe realizar tratamiento con frmacos anti
vitamina K (Acenocumarol) durante el embarazo.

Linfoma de Zona Marginal tipo MALT.


Linfoma Linfoplasmactico .
Linfoma Difuso de Clulas Grandes B.
Linfoma de Clulas del Manto.
Linfoma Linfoblstico de clulas B precursoras.

102. Anciana que llevan sus vecinos a urgencias porque la ven algo atontada y con manchas de deposicin en la ropa. TA 100/60 mmHg, FC 100 lpm;
sentada TA 70/30 mmHg, FC 105 lpm. PVY
normal. Respiracin de Kussmaul. No focalidad
neurolgica. Peso de 50 kg. Lab: pH 7.25, PCO2
14 mmHg, Bicarbonato 5 mg/dl, Na 133 mEq/L,
k 2.5 mEq/L, Cl 118 mE/L, Cr 3.4 mg/dl, NUS
60, Prot 8 g/dl. Cal de las siguientes respuestas
es correcta?

La alteracin gentica caracterstica del linfoma


de Burkitt es la traslocacin t(8;14). Qu oncogn se activa mediante esta traslocacin?

1.
2.
3.
4.
5.

BCL-2.
c-MYC.
Ciclina D1.
MAF.
BCL-6.

1. El trastorno cido-base que presenta es un


acidosis respiratoria.
2. La compensacin para corregir la acidosis no
es adecuada.
3. Con esta exploracin descartamos que est
deshidratada.
4. En ningn caso debemos ponerle bicarbonato.
5. Tiene una insuficiencia renal aguda de causa
prerrenal.

100. De todas las siguientes, cal es la complicacin


que puede observarse en los enfermos que reciben heparina?

1.
2.
3.
4.
5.

Insuficiencia renal aguda.


Anemia hemoltica autoinmune.
Trombosis venas suprahepticas.
Sndrome leucoeritroblstico.
Plaquetopenia.

103. Cual de las siguientes alteraciones no es caracterstica del hiperparatiroidismo secundario de los
pacientes con insuficiencia renal:

1.
2.
3.
4.
5.

101. Mujer de 25 aos de edad que acude a consulta


porque desea quedar embarazada y quiere saber
qu tratamiento debe realizar durante el eventual embarazo, ya que es portadora del Factor V
Leiden en heterocigosis. Nunca ha tenido ningn
fenmeno trombtico. Se realiz la determinacin del mencionado factor como estudio familiar tras un episodio de embolia del pulmn en
un hermano. Qu tratamiento se debe aconsejar?

Hiperfosfaturia.
Calcificaciones vasculares.
Reabsorcin subperistica.
Producto fosfo-clcico elevado.
Aumento de las fosfatasas alcalinas.

104. Existen mltiples factores que contribuyen a la


afectacin renal en el Mieloma Mltiple y causante de insuficiencia renal. De las cinco respuestas slo una es falsa.

1. Dado que la gestacin es un estado protrombtico, existira alto riesgo de tromboembolia


venosa por lo que se debe desaconsejar el
embarazo.
2. Se debe realizar tratamiento con heparina de
bajo peso molecular a dosis profilcticas en
el puerperio inmediato siendo opcional realizar igual tratamiento durante el embarazo.

1. Proteinuria de Bence-Jones y tubulopata con


cilindros.
2. Amiloidosis.
3. Vasculitis.
4. Depsitos de cadenas ligeras.
5. Hipercalcemia e hiperuricemia.

- 18 -

105. Cal de los siguientes factores NO favorece el


desarrollo de pielonefritis aguda?

1.
2.
3.
4.
5.

109. La afectacin renal neoplsica metastsica ms


frecuente es por:

Nefrolitiasis.
Reflujo vsico-ureteral.
Rin de herradura.
Quiste renal simple.
Tumor de urotelio.

1.
2.
3.
4.
5.

106. Seale cal de las siguientes afirmaciones referentes al Sndrome de Secrecin Inadecuada de
ADH (SIADH) es FALSA:

110. De los posibles mecanismos de produccin de


incontinencia urinaria tras una prostatectoma
radical, seale la respuesta FALSA:

1. Los pacientes con SIADH tienen una eliminacin de sodio por orina inferior a 40
mEq/litro como consecuencia del reajuste a
un nivel ms bajo de osmolaridad ("reset
osmotat").
2. El dolor postoperatorio y las enfermedades
neuropsiquitricas son etiologas conocidas
del SIADH.
3. El SIADH constituye en nuestros das una de
las causas ms frecuentes de hiponatremia
normovolmica.
4. El SIADH se caracteriza por una hiponatremia hipoosmtica con una osmolaridad urinaria mayor de 100 mosm/kg.
5. La paroxetina y la sertralina deben ulizarse
con precaucin, por ser frmacos causantes
de SIADH en algunos pacientes.

1.
2.
3.
4.
5.

Baja acomodacin vesical.


Afectacin contrctil del detrusor.
Hiperactividad del detrusor.
Disinergia vesico-esfinteriana.
Dficit esfinteriano.

111. Un paciente varn de 45 aos de edad con antecedentes de litiasis rica con clicos nefrticos
expulsivos de repeticin desde hace 25 aos,
acude a Urgencias refiriendo palpitaciones y
dolor lumbar derecho intenso desde hace 2 horas
antes. El anlisis de orina muestra un pH de 5.5
y leucocituria sin proteinuria. El electrocardiograma confirma una fibrilacin auricular no
conocida previamente. En la bioqumica sangunea destaca una creatinina de 0.9 mg/dl, un calcio de 11 mg/dl y una LDH de 950 U/l. Cal es
la maniobra diagnstica ms til a realizar?.

107. Mujer de 19 aos, peso 60 kg, con deshidratacin aguda por larga exposicin al sol. Presin
arterial tumbada 100/60 mmHg. De pie, 70/50
mmHg con sensacin de mareo. Niveles de sodio
srico 155 mmol/L. Cal es el tratamiento ms
correcto, en las primeras 24 h teniendo en cuenta
la totalidad de los datos de que disponemos?

1.
2.
3.
4.
5.

1. Suero salino hipertnico (3%), 500 ml +


500ml de glucosado de 5%.
2. Suero hiposalino (0.45%), 3000 ml.
3. Suero glucosado 5%, 1000 ml.
4. Hidratacin oral con 1 litro de agua.
5. Suero salino isotnico (0.9%), 2000 ml.

Radiografa simple de abdomen.


Urografa intravenosa.
TC espiral con contraste.
Ecografa abdomal.
Ecocardiografa doppler.

112. Un varn de 64 aos consulta por fiebre tos


disnea y dolor pleurtico derecho de 1 semana de
evolucin. En la radiografa de trax se aprecia
un derrame pleural derecho loculado que ocupa
dos terceras partes del hemitrax. Durante la
toracocentesis se extrae un lquido amarillento
cuyo anlisis muestra: leucocitos 15.000/L, 92%
de neutrfilos, glucosa 30 mg/dL, pH 7, lactato
deshidrogenasa 3500 U/L, adenosina desaminasa
45 U/L y ausencia de grmenes en la tincin de
Gram. Cul es la siguiente accin ms apropiada en este paciente?

108. El tratamiento de eleccin para el carcinoma in


situ de vejiga es:

1.
2.
3.
4.
5.

Metstasis de cncer de prstata.


Metstasis de rin contralateral.
Invasin por clulas neoplsicas de linfomas.
Metstasis de tumor germinal.
Invasin por clulas neoplsicas del pulmn.

Radioterapia conformada.
Radioterapia convencional.
Administracin de BCG intravesical.
Quimioterapia sistmica.
Quimioterapia local.

1. Antibioticoterapia intravenosa.
- 19 -

2. Antibioticoterapia intravenosa y repetir la


toracocentesis diagnstica en 24 horas.
3. Antibioticoterapia intravenosa y repetir la
toracocentesis diagnstica si no hay mejora
en 48 horas.
4. Antibioticoterapia intravenosa y realizar una
toracocentesis evacuadora (teraputica) si se
aisla algn grmen en el cultivo del lquido
pleural.
5. Antibioticoterapia intravenosa y colocar un
tubo o catter torcico para drenar todo el
lquido pleural.

recibi tratamiento con amoxicilina, presentando posteriormente un exantema cutneo maculoso generalizado. Se realiza analtica que presenta
ligera leucocitosis con linfocitosis y presencia de
linfocitos activados, ligera trombopenia y transaminasas levemente aumentadas. Cul sera el
diagnstico ms probable de este cuadro clnico?

1. Es un cuadro tpico de mononucleosis infecciosa.


2. Infeccin por virus varicela zoster.
3. Toxoplasmosis aguda.
4. Enfermedad de Lyme.
5. Infeccin por herpes virus 8.

113. Una de las siguientes condiciones no es definitoria de SIDA:

1.
2.
3.
4.
5.

117. Seale cul de las siguientes afirmaciones referidas al paludismo o sus agentes causales no es
cierta.

Candidiasis oral.
Toxoplasmosis cerebral.
Tuberculosis pulmonar.
Neumona por Pneumocystis jiroveci.
Linfoma cerebral primario.

1. La infeccin del ser humano comienza con la


picadura del mosquito Anopheles, que introduce esporozoitos con su saliva en el sistema
circulatorio.
2. Plasmodium vivax es selectivo en cuanto a
que solo invade hemates jvenes inmaduros.
3. Plasmodium ovale se encuentra sobre todo en
Africa tropical.
4. La afectacin del cerebro (paludismo cerebral) es ms frecuente en la infeccin por
Plasmodium falciparum.
5. La deteccin de una infeccin mixta no tiene
influencia directa sobre el tratamiento elegido.

114. En la relacin con las pautas de tratamiento de


la Tuberculosis en pacientes con Infeccin por el
Virus de la Inmunodeficiencia Humana (VIH),
cal de las siguientes respuestas es la correcta?.

1. Son ms prolongadas que en los pacientes sin


VIH.
2. Deben incluir ms frmacos que en los pacientes sin VIH.
3. Son las mismas que en los pacientes sin VIH.
4. No presentan interacciones farmacolgicas
con los antirretrovirales.
5. El tratamiento debe ser indefinido.

118. Mujer de 47 aos de edad. Natural de Cochabamba (Bolivia), reside en Espaa desde hace 8
aos. A medidados del ao 2009 comienza con
palpitaciones, disnea y edemas progresivos en
extremidades inferiores. Es diagnosticada de
insuficiencia cardaca secundaria a miocardiopata por Chagas. Cul es es agente etiolgico de
esta enfermedad endmica en varios pases de
Latinoamrica?

115. A un joven asintomtico de 14 aos cuyo padre


acaba de ser diagnosticado de Tuberculosis Pulmonar se le realiza un Mantoux, con resultado
negativo. Qu actitud es la adecuada?

1. Tranquilizarle, ya que no tiene ningn riesgo.


2. Realizarle una radiografa de trax.
3. Iniciar quimioprofilaxis y repetir la prueba
cutnea a los 3 meses.
4. Repetir la prueba cutnea al mes.
5. Habra que realizar estudio del esputo antes
de iniciar quimioprofilaxis con Isoniacida
durante 1 ao.

1.
2.
3.
4.
5.

Toxoplasma gondii.
Trypanosoma brucei.
Leishmania donovani.
Giardia lamblia.
Trypanosoma cruzi.

119. La sepsis grave se define como la sepsis que presenta uno o ms signos de disfuncin de rganos.
Entre los siguientes seale el incorrecto.

116. Mujer de 17 aos de edad que acude a urgencias


por un cuadro agudo de fiebre elevada, dolor
farngeo y adenopatas cervicales. Previamente
haba sido diagnosticada de faringitis aguda y

1. Hipotensin.
- 20 -

2. Oliguria (diuresis <0.5 ml/kg).


3. Niveles plasmticos elevados de protena C
reactiva o procalcitonina.
4. Hipoxemia con Pa 02/Fi 02 < 250.
5. Hiperlactoacidemia.

intensidad es directamente proporcional a la


eficacia antitumoral, seale cual:

1.
2.
3.
4.
5.

120. Un varn de 19 aos consulta por un cuadro de


febrcula, mialgias y odinofagia, acompaado de
mculas cutneas generalizadas no pruriginosas
durante los ltimos 7 das. En el interrogatorio
refiere, como nico suceso mdico, haber presentado una lcera anal que le causaba dolor a la
defecacin en el mes anterior que desapareci
progresivamente sin ninguna intervencin.
Cul es el germen causante de este cuadro clnico?

1.
2.
3.
4.
5.

124. Hombre de 60 aos que acude a urgencias por


crisis comicial. Ex-fumador desde hace 3 aos,
sin otros antecedentes de inters. Una TAC evidencia metstasis mltiples. Cal es el origen
ms probable?

1.
2.
3.
4.
5.

Treponema pallidum.
Gonococo.
Virus de herpes simple tipo 2.
Chlamydia trachomatis.
Entamoeba coli.

Cncer de cabeza y cuello.


Cncer de pulmn.
Cncer de vejiga urinaria.
Cncer de colon.
Cncer de pncreas.

125. El psiquiatra de interconsulta acude a examinar


a un paciente en Cardiologa que rechaza un
tratamiento que el facultativo a cargo del caso
considera indicado e insustituible. Tras la exploracin no aprecia patologa psiquitrica alguna;
el paciente comprende la informacin que se le
ha aportado, los posibles beneficios y riesgos del
tratamiento (includo el fallecimiento), y sopesa
adecuada y racionalmente esta informacin,
pero se mantiene firme en su decisin de no ser
tratado. El psiquiatra concluye que el enfermo
est capacitado para la toma de decisiones en el
mbito de la atencin sanitaria. La enfermedad
que presenta el paciente, que se niega a firmar el
alta voluntaria, no es infecciosa ni representa
ningn riesgo especial para terceros. Cul es la
decisin ms adecuada a tomar?

121. La funcin ms importante de las isoformas


FGF-1 y FGF-2 de los factores de crecimiento
fibroblstico (FGFs) es:

1. Estimular la sntesis de colgeno y la proliferacin de los fibroblastos.


2. Estimular la sntesis de colgeno y la proliferacin de los queratinocitos.
3. Estimular la sntesis de colgeno y la migracin de los queratinocitos.
4. Estimular la angiognesis.
5. Estimular la epitelizacin.

122. Una mujer de 30 aos con antecedentes personales de drogadiccin en periodo actual de desintoxicacin es trada a Urgencias con un cuadro
de profunda somnolencia, sin otra sintomatologa. Logramos averiguar que en su tratamiento
se incluyen olanzapina y benzodiacepinas. Qu
es lo que debemos hacer en primer lugar:

1.
2.
3.
4.
5.

Hipertensin arterial.
Ictus.
Hemorragia.
Infarto agudo de miocardio.
Rash cutneo acneiforme.

1. Ingreso en Psiquiatra para trabajar la negacin y la rabia inherentes a su proceso.


2. Solicitar la intervencin de familiares que
autoricen la intervencin.
3. Alta hospitalaria.
4. Incapacitacin civil.
5. Informar al juez con el fin de que ordene
tratamiento.

Proceder a intubacin endotraqueal.


Realizar una radiografa de crneo.
Administrar flumazenilo.
Administrar naloxona.
Esperar a que despierte.

126. De las siguientes afirmaciones sobre el control


sintomtico en pacientes al final de la vida seale
la correcta:

123. Cetuximab es un anticuerpo monoclonal antiEGFR empleado en el tratamiento del cncer


colorrectal izquierdo avanzado. Existe un efecto
secundario caracterstico de este frmaco cuya
- 21 -

1. Si el paciente tiene dolor se emplearn siempre opiceos potentes para control sin tener
en consideracin la severidad y caractersticas del dolor.
2. Si precisamos sedacin para control de la
fase agnica podemos realizarla de forma
exclusiva con morfina.
3. Los antidepresivos tricclicos no se consideran medicacin coadyuvante para el alivio
sintomtico del dolor.
4. Los neurolpticos a dosis bajas ayudan al
control de las nuseas desencadenadas por el
inicio de un tratamiento con opiceos.
5. El dolor de caractersticas viscerales es el que
peor respuesta tiene al tratamiento con opiceos.

confusa de lo normal. La paciente est diagnosticada de enfermedad de Alzheimer en grado moderado tiene adems fibrilacin auricular, depresin y artrosis. Su tratamiento estable desde
hace 3 aos incluye digoxina, acenocumarol,
fluoxetina desde hace 4 meses e ibuprofeno desde
hace un mes por dolores articulares. A la exploracin su tensin arterial es de 130/80 mm de
Hg, la frecuencia cardiaca est en 48 lpm y la
respiratoria en 18 por min. Cul de los siguientes es la causa ms probable del aumento de la
confusin?

1.
2.
3.
4.
5.

127. Cal es el tratamiento de la obstruccin intestinal por carcinomatosis peritoneal en la fase terminal de la enfermedad?

Progresin de la enfermedad de Alzheimer.


Empeoramiento de la depresin.
Intoxicacin digitlica.
Demencia por cuerpos de Lewy.
Toxicidad por Ibuprofeno.

130. Un hombre de 64 aos de edad con antecedentes


de HTA y cardiopata isqumica acude a urgencias por aparicin de dolor torcico opresivo
mientras vea la televisin. En la consulta de
clasificacin (Triage) de urgencias se detectan
cifras de presin arterial 155/95 mm Hg y una
saturacin capilar de O2 de 95%. Cul es la
conducta ms correcta entre las que a continuacin se citan?

1. Sonda nasogstrica, aspiracin continua,


sueroterapia intravenosa.
2. Ciruga desobstrutiva.
3. Alimentacin parenteral total y laxantes.
4. Administracin por va subcutnea de morfina, buscapina, haloperidol y octetrido.
5. Dieta absoluta y sueros por va intravenosa o
subctanea.

1. Indicarle que vaya a la sala de espera. Le


avisarn para la realizacin de pruebas.
2. Este paciente debe ser atendido en el box de
emergencia vital (cuarto de shock).
3. Avisaremos al cardilogo de guardia para la
valoracin del paciente.
4. Desde la consulta de Triage se le remitir a la
unidad coronaria.
5. Haremos un ECG en menos de 10 minutos.

128. A un hombre de 74 aos de edad con historia de


enfermedad coronaria se le ha practicado un
triple by-pass coronario hace 3 das. Se le extub
en el primer da postoperatorio. Ahora requiere
valoracin porque est agitado a pesar de antipsicticos y sujeccin mecnica. Se quita el tubo
de oxgeno a pesar de sujeccin en muecas.
Debido a su confusin se ha limitado la deambulacin y mantiene una sonda vesical. Los signos
vitales son normales y el mdico recomienda
quitar la sonda vesical y realizar una anlisis de
orina con cultivo. Cul de los siguientes puede
significativamente disminuir la agitacin y ayudar a resolver el estado confusional agudo?

131. Una paciente de 53 aos a la que hemos atendido


por un dolor de rodilla -orientado como artrosis- entra a la consulta de forma brusca y de pie
y en tono spero exclama: "Estoy enfadado con
usted! Hay que ver! Lo que me dio no me ha
hecho nada estoy en un grito!". La mejor intervencin nuestra es:

1. Limitar la movilidad a transferencia al silln


hasta que se resuelva el estado confusional
agudo.
2. Retirar las sujecciones mecnicas de las muecas.
3. Colocar al paciente en una silla geritrica al
lado del control de enfermera.
4. Ayudarle en la ingesta.
5. Aumentar la dosis de antipsicticos.

1. Haga el favor de salir y vuelva usted a entrar


en la consulta de manera educada.
2. Tome asiento y ver qu puedo hacer por
usted.
3. Esta medicacin que le di es la mejor y ms
segura para la artrosis de rodilla.
4. No sabe como lo siento, a veces ocurre, lo
lamento de veras.

129. La hija de una anciana de 82 aos nos consulta


porque ha encontrado a su madre mucho ms
- 22 -

5. Yo tambin estoy enfadado con usted por la


forma en que me chilla, si me habla como
persona seguro que nos entenderemos.

4. Requiere un cateterismo cardaco y la angiografa son imprescindibles.


5. Una RX Trax, el E.C.G y la Ecocardiografa
2D.

132. Los familiares de un paciente recientemente


dado de alta de un hospital solicitan al Servicio
de Atencin al Usuario del mismo la historia
clnica completa de un paciente, sin especificar el
motivo. Cul es la actitud adecuada?

135. Un recin nacido de 37 semanas de edad gestacional, sin hallazgos patolgicos en la ecografa
prenatal, presenta en la exploracin realizada en
el paritorio un "stop" al paso de la sonda nasogstrica. La radiografa de trax y abdomen
muestra un bolsn esofgico atrsico, con una
neumatizacin gastrointestinal normal. Tras
evalucin diagnstica que descarta otras anomalas y encontrndose en situacin respiratoria
estable, se decide intervencin quirrgica. En
qu se basa la prioridad de intervenir a este
paciente?

1. El hospital no la facilitar porque la historia


clnica de un paciente no puede salir del
centro.
2. Los familiares no la pueden solicitar. Lo
tiene que hacer el propio paciente.
3. El hospital tiene que facilitarla sin condiciones porque el propietario de la historia clnica
es el paciente.
4. Se facilitar el informe de todas las exploraciones complementarias y los comentarios
generados por los mdicos responsables
cuando stos lo consideren oportuno.
5. Es necesario un requerimiento judicial para
facilitar la informacin de una historia clnica.

1. Por la imposibilidad de deglutir la saliva.


2. Por las malformaciones cardiacas frecuentemente asociadas.
3. Por el riesgo de neumonitis por aspiracin.
4. Por la imposibilidad de alimentacin enteral.
5. Por la traqueomalacia asociada.

136. Nio de 1 mes, alimentado con lactancia materna exclusiva, consulta porque realiza una deposicin cada 5 6 das con esfuerzo pero de consistencia blanda, cul sera nuesta actitud?

133. Una nia de 13 aos de edad, nos consulta por


presentar prdida de peso, astenia, amenorrea
secundaria y estreimento. En la exploracin
destaca una temperatura de 35C, un peso inferior al percentil 3, bradicardia e hipotensin
ortosttica. En la analitica se comprueba anemia
con leucopenia leve, colesterol de 230 mg/dl y en
el electrocardiograma una onda T aplanada con
descenso del ST y QTc prolongado. Qu proceso sospechara y que hara acontinuacin?

1. Iniciar tratamiento laxante.


2. Suplementar la lactancia materna con frmula antiestreimiento.
3. Indicar estimulacin rectal diaria.
4. Se considera un ritmo intestinal normal.
5. Se deriva a Digestivo Infantil para descartar
megacolon aganglinico.

1. Sospechara una anorexia nerviosa e indagara acerca de su ingesta, imagen corporal, etc.
2. Sospechara un hipotiroidismo y solicitara
TSH y hormonas tiroideas.
3. Sospechara una miocardiopata y solicitara
una ecocardiografa.
4. Sospechara una enfermedad celiaca y solicitara anticuerpos anti transglutaminasa IgA.
5. Sospechara un panhipopituitarismo y solicitara una TC craneal.

137. Nia de 12 aos de edad, hija nica de padres


sanos no consanguneos. No refieren antecedentes personales ni familiares de inters. En el
examen clnico para iniciar una actividad deportiva, usted encuentra una discreta hepatomegalia
de consistencia normal, por lo que solicita una
analtica, en la que destaca una ASAT de 80 U/L,
ALAT 105 U/L, proteinograma electrofortico
con todas las fracciones protenas en rango normal y negatividad de la serologa de virus hepatotropos. Qu debera descartar y qu prueba
indicara para ello?

134. Para confirmar la sospecha clnica de cardiopata congnita en un nio, usted solicitara:

1. La realizacin de una TC o RM.


2. Basta con la anamnesis y exploracin clnica.
3. Es necesario realizar un ECG de 24 horas
(Holter).

1. Dficit de alfa 1 antitripsina. TAC torcico


para confirmar enfisema.
2. Mucoviscidosis. Deteminacin de cloro en
sudor.
3. Hepatitis autoinmune. Biopsia heptica.
- 23 -

4. Glucogenosis tipo VI (dficit d fosforilosa).


Biopsia muscular.
5. Enfermedad de Wilson. Determinar ceruloplasmina y cobre en sangre y orina.

1.
2.
3.
4.
5.

138. Se define la pubarquia prematura como la aparicin de vello pubiano antes de los 8 aos en la
nia y antes de los 9 en los nios. En relacin
con esta entidad, indique la respuesta verdadera:

Meningitis vrica.
Meningitis bacteriana.
Meningitis tuberculosa.
Sndrome mononuclesico.
Sndrome de Guillain Barr.

141. La causa ms frecuente de sndrome de dificultad respiratoria o enfermedad de membrana


hialina en el recin nacido prematuro es:

1. El defecto de surfactante pulmonar por inmadurez.


2. Un neumotrax a tensin.
3. Un sndrome de aspiracin meconial.
4. Una infeccin respiratoria.
5. La escasa fuerza muscular.

1. La causa ms frecuente es la hiperplasia


suprarenal congnita.
2. Es el signo inicial ms habitual de pubertad
precoz central en la nia.
3. Se asocia a mayor incidencia de hiperandrogenismo en la edad postpuberal.
4. En la mayora de los casos requiere tratamiento.
5. Se acompaa de una edad sea retrasada.

142. Con respecto al sndrome nefrtico idioptico,


seale la respuesta correcta:

139. Nio de 2 aos, en sus antecedentes personales


destacan: 3 episodios de otitis media aguda, 1
meningitis meningoccica y 2 neumonas (una de
lbulo medio y otra de lbulo superior izquierdo). Ha ingresado en 3 ocasiones por prpura
trombopnica (en tres ocasiones los anticuerpos
antiplaquetas fueron negativos y en la mdula
sea se observaban megocariocitos normales).
Varios varones de la familia materna haban
fallecido en la infancia por procesos infecciosos.
En la exploracin fsica presenta lesiones tpicas
de dermatitis atpica. En el estudio inmunolgico destaca una leve disminucin de subpoblaciones de linfocitos T; elevacin de IgA e IgE; disminucin de IgM y IgG en el lmite inferior de la
normalidad.

1. El tipo histolgico ms habitual es la hiperplasia mesangial difusa.


2. El sndrome nefrtico idioptico por lesiones
mnimas (sndrome nefrtico de cambios
mnimos) es ms frecuente en los nios mayores de 8 aos.
3. La glomeruloesclerosis segmentaria y focal
es la forma histolgica que mejor responde al
tratamiento con corticoides.
4. La biopsia renal es necesaria en todos los
pacientes, para poder establecer una orientacin pronstica e instaurar el tratamiento
adecuado.
5. La dislipemia y la hipercoagulabilidad sangunea son manifestaciones clnicas habituales en los pacientes con sndrome nefrtico.

Cal es el diagnstico ms probable?

143. Chico de 12 aos obeso (87 kg). Refiere desde


hace 4 meses dolor mcanico en muslo y rodilla
derecha. A la exploracin clnica se evidencia
una discrepancia de la longitud de las extremidades inferiores de 1 cm y actitud de la extremidad inferior derecha en rotacin externa con
prdida de la rotacin interna. Debemos sospechar que presenta:

1. Sndrome de Wiskott-Aldrich.
2. Sndrome hiper IgE.
3. Hipogammaglobulinemia transitoria de la
infancia.
4. Inmunodeficiencia combinada severa ligada
al X.
5. Inmunodeficiencia variable comn.

1.
2.
3.
4.
5.

140. Ante un nio de 7 meses con fiebre e irritabilidad, fontanela abombada y un estudio de lquido
cefalorraqudeo con 110 clulas/mm3 (75% linfocitos), protenas 120 mg/dl y glucosa 28 mg/dl
(glucemia srica 89 mg/dl), cul es la sospecha
diagnstica ms razonable?

Artritis crnica juvenil.


Epifisolisis capitis femoris.
Fractura del cuello femoral por sobrecarga.
Enfermedad de Perthes.
Secuelas de displasia del desarrollo de la
cadera.

144. Paciente de 67 aos con antecedente de meno- 24 -

pausia a los 55 aos, 3 gestaciones con 3 partos


eutcicos, diabetes tipo 2 de 6 aos de evolucin,
tratamiento con nifedipino por hipertensin
arterial. Consulta por cuadro de metrorragia
escasa intermitente de 2 meses de evolucin. En
la exploracin ginecolgica se aprecian unos
genitales externos sin lesiones, un crvix uterino
de aspecto atrfico, un tero y anejos normales a
la palpacin y un estudio citolgico de crvix
normal. En el estudio ecogrfico transvaginal se
aprecia un endometrio hiperecognico de 7 mm.
Cul de las siguientes pruebas es la ms indicada y con mayor sensibilidad para establecer un
diagnstico?

1.
2.
3.
4.
5.

148. Ante una mujer de 59 aos que consulta por


secrecin hemtica por el pezn derecho (unilateral), espontnea y uniorificial sin nodularidad
palpable. Cul es el diagnstico ms probable?

1. Dado el carcter hemtico de la secrecin el


diagnstico ms probable es el de carcinoma
ductal infiltrante.
2. Papiloma intraductal.
3. Enfermedad de Paget del pezn.
4. Hiperprolactinemia tumoral.
5. La mamografa nos indicar el diagnstico.

Conizacin de crvix.
Citologa endometrial.
Histeroscopia y biopsia de endometrio.
Resonancia nuclear magntica de pelvis.
Exploracin bajo anestesia de aparato genital
y biopsia de crvix y endometrio.

149. Una mujer de 32 aos asintomtica consulta


para una revisin en salud ginecolgica porque
desea quedarse embarazada. En dicha revisin
se le detecta un mioma uterino de 4 cm en parte
intramural y en parte subseroso situado en la
cara anterior del tero y que no deforma la cavidad endometrial. Qu actitud le recomendara?

145. El estudio WHI (Momens Health Initiative) puso


de manifiesto que la terapia de reemplazo con
estrgeno/progesterona oral en la menopausia
produce mayor riesgo de padecer:

1.
2.
3.
4.
5.

1. Miomectoma por va laparoscpica.


2. Miomectoma por va laparotmica.
3. Embolizacin del mioma por cateterismo
arterial .
4. Tratamiento con anlogos de Gn-RH durante
tres meses antes de intentar el embarazo.
5. Intentar el embarazo sin ningn tratamiento
previo.

Enfermedades vasculares.
Diabetes mellitus.
Aumento del colesterol LDL.
Carcinoma colo-rectal.
Sntomas vasomotores.

150. La ciruga histeroscpica permite el tratramiento de una gran variedad de patologas intrauterinas, excepto:

146. Una mujer de 42 aos con implantes mamarios


consulta por tumor palpable en cuadrantes internos de mama derecha. Qu prueba de imagen debe indicarse para descartar un cncer?

1.
2.
3.
4.
5.

1.
2.
3.
4.
5.

Ecografa mamaria.
Mamografa.
Resonacia Magntica.
TAC.
PET.

Los plipos endometriales.


El embarazo tubrico.
Los miomas submucosos.
Algunas malformaciones.
Las hemorragias por hipertrofia endometrial.

151. Selese cul de las siguientes formas neoplsicas del ovario no pertenece al grupo de los tumores del epitelio-estroma:

147. La principal causa de aparicin del cncer de


cuello uterino es:

1.
2.
3.
4.
5.

1. Tabaco.
2. Infeccin por ciertas cepas de virus del papiloma humano.
3. Multiparidad.
4. Inmunosupresin.
5. Antecedente de displasia vaginal.

Carcinoma seroso.
Tumor endometrioide.
Carcinoma de clulas claras.
Carcinoide.
Tumor de Brenner.

152. Paciente de 76 aos de edad, paridad 3-0-1-3,


con menopausia a los 52 aos. Relata que desde
- 25 -

2. En la forma HEBEFRNICA existe predominio de alteraciones psicomotrices, negativismo y conductas esteriotipadas.


3. La presencia de estados de exaltacin anmica, maniforme con estravagancias es propia
de las formas SIMPLES.
4. La forma PARANOIDE presenta predominantemente trastornos del pensamiento y
trastornos sensoperceptivos.
5. Los sntomas obsesivos y compulsivos asociados a disgregacin progresiva son habituales en la forma ESQUIZOTPICA.

hace al menos 4-5 aos presenta prurito vulvar


de intesidad variable que ha sido tratado algunas
veces mediante automedicacin y otras por indicacin de su mdico generalista con preparados
tpicos (cremas y lavados). El prurito ha evolucionado as con intermitencias pero desde hace
3-4 meses nota adems una pequea tumoracin
en el labio mayor izquierdo de la vulva y de la
que se producen prdidas serohemticas al roce
desde hace unos das; por ello consulta al gineclogo. Al interrogatorio refiere disuria ocasional y
el estado general es bueno. Cul es el diagnstico ms probable en esta paciente?

1.
2.
3.
4.
5.

156. Una mujer de 23 aos acude a urgencias por


encontrarse muy nerviosa tras una discusin con
su pareja. En su historia clnica se reflejan varias demandas semejantes en el ao anterior, en
dos de ellas tras un gesto autoltico. Se constatan
tambin conflictos frecuentes en las relaciones de
pareja, cambios laborales y discusiones familiares. Dice sentirse incomprendida por todos incluidos los psiquiatras que la atienden. El diagnstico es:

Herpes genital.
Enfermedad de Paget de la vulva.
Carncula uretral.
Granuloma crnico de la vulva.
Carcinoma vulvar escamoso.

153. De los siguientes marcadores ecogrficos, seale


el que no se considera para el diagnstico prenatal del primer trimestre de la gestacin:

1.
2.
3.
4.
5.

1.
2.
3.
4.
5.

Sonoluscencia nucal.
Hueso nasal.
Regurgitacin tricuspdea.
Quiste del plexo coroideo.
Estudio del flujo en el ductus venoso.

Trastorno de la personalidad lmite.


Trastorno de la personalidad histrinico.
Distimia.
Trastorno disociativo.
Trastorno de despersonalizacin.

157. La coexistencia de alucinaciones y de alteraciones de la conciencia es sugestiva de:

154. Cundo NO est indicado hacer profilaxis antibitica durante el parto para la prevencin de la
infeccin neonatal por estreptococo del grupo
B?.

1.
2.
3.
4.
5.

1. En la embarazada con screning positivo en


vagina o recto en las 5 semanas previas al
parto.
2. En la embarazada con el antecedente de un
recin nacido con infeccin por el estreptococo del grupo B.
3. En la embarazada con bacteriuria por estreptococo en el embarazo en curso.
4. En la embarazada con cultivo desconocido o
no realizado y parto pretrmino.
5. En la cesrea programada en la gestante sin
trabajo de parto y con membranas ntegras.

Los estados ansiosos.


Los estados confusionales.
Los estados depresivos.
Los estados manacos.
Los estados psicticos.

158. Una estudiante universitaria de 19 aos acude a


la consulta acompaada por sus padres refiriendo sentirse en los ltimos dos meses progresivamente ms astnica, con prdida de apetito y de
peso y con mayores dificultades para concentrarse en los estudios. En la anamnesis tambin
destaca que ha perdido inters en salir con las
amigas, presenta ideas de muerte sin ideacin
autoltica y cogniciones pesimistas de futuro. Su
peso es el 90% del considerado ideal por edad y
gnero. No presenta fobia ponderal ni distorsin
de la imagen corporal. El diagnstico ms adecuado es:

155. Referido a las formas clnicas de la esquizofrenia


cul de las siguientes afirmaciones es cierta?

1. La forma CATATNICA se presenta con


sintomatologa pobre, predominio de delirios
y suele ser de inicio temprano.

1. Anorexia nerviosa.
2. Trastorno de ansiedad.
3. Trastorno lmite de la personalidad.
- 26 -

4. Distimia.
5. Trastorno depresivo mayor.

2.
3.
4.
5.

159. Mujer de 52 aos que acude a urgencias con una


parlisis bilateral de las extremidades superiores
que se haba instaurado de manera brusca dos
horas antes. En la entrevista se muestra risuea
y verbaliza no entender por qu sus familiares
estn tan preocupados. La familia refiere que el
da anterior su hijo haba sido detenido por la
polica por trfico de drogas y todava no haba
podido contactar con l. Cul de los siguientes
es el diagnstico ms probable?

1.
2.
3.
4.
5.

Haloperidol.
Sertralina.
Metilfenidato.
Clozapina.

163. Cal es la causa que con mayor frecuencia


produce los ndulos de las cuerdas vocales?

1.
2.
3.
4.
5.

Trastorno facticio.
Trastorno delirante.
Hipocondriasis.
Trastorno depresivo mayor.
Trastorno conversivo.

La tuberculosis larngea.
La amiloidosis larngea.
Las infecciones virales.
La infeccin por haemphilus infuenzae.
Las alteraciones funcionales de la voz.

164. Paciente de 64 aos fumador y bebedor importate sin otros antecedentes de inters. El cuadro
que explica es de odinodisfagia de 3 meses de
evolucin y otalgia izquierda. Aporta informe
del odontlogo que descarta causa de origen
dental. Tambin refiere haber sido visitado por
diferentes especialistas aportando un TAC de
cabeza y cuello (sin contraste yodado) que es
informado como normal. A la exploracin destaca una disminucin en la capacidad de propulsin de la lengua; la palpacin de la base de lengua izquierda objetiva una induracin de consistencia ptrea de aprox 3cm y con laringoscopia
indirecta no se observa ninguna ulceracin de la
mucosa. La palpacin cervical es negativa para
adenopatas. Cal es su diagnstico de presuncin y conducta?.

160. Una de las siguientes caractersticas clnicas


corresponde al Trastorno Delirante:

1. Es un delirio estructurado y creible, formado


a base de creencias incorrectas acerca de la
realidad externa.
2. Trastornos del curso del pensamiento (bloqueo o disgregacin del curso del pensamiento).
3. Sntomas negativos como abulia y aplanamiento afectivo.
4. Alucinaciones auditivas.
5. Existe una conciencia clara de enfermedad.

1. Causa dental / revaloracin por el odontlogo.


2. Absceso de la base de la lengua / desbridamiento.
3. Anquiloglosia / liberacin quirrgica de la
misma.
4. Proceso maligno de orofaringe / repetir TC y
biopsia.
5. Inflamacin inespecfica / tratamiento corticoideo.

161. El litio constituye el tratamiento profilctico de


primera eleccin en el Trastorno Bipolar tipo I.
cul de los siguientes efectos secundarios es
caracterstico de este frmaco?.

1. Temblor postural fino.


2. Cefaleas de predominio frontal.
3. Atracones bulmicos especialmente de hidratos de carbono.
4. Hiperlipemia (aumento de trigliceridos en
sangre).
5. Puede inducir depresin con riesgo autoltico
especialmente importante.

165. La necrosis de los maxilares est relacionada con


la administracin de:

1.
2.
3.
4.
5.

162. Seale cal de los siguientes tratamientos est


indicado para el trastorno por dficit de atencin con hiperactividad.

Bifosfonatos.
Terramicina.
Cisplatino.
Calcio.
Corticoides.

166. Nio que acude a consulta de ambulatorio porque se lleva la mano al odo, sin fiebre ni signos

1. Fluoxetina.
- 27 -

1.
2.
3.
4.
5.

inflamatorios agudos y que en la exploracin se


observa un tmpano sonrosado/ambarino. Como
antecedentes refiere que hace un mes present
un episodio de otalgia, fiebre acompaada de
irritabilidad y que fue tratado con amoxicilina
40 mg/kg/da. Cual es el diagnstico mas probable?

1.
2.
3.
4.
5.

170. Paciente que presenta desde hace 2 semanas una


erupcin pruriginosa constituida por mltiples
ppulas eritemato-violaceas poligonales, de superficie aplanada, localizadas preferentemente
en cara anterior de muecas, zona pretibial y
rea lumbar. Adems presenta lesiones reticuladas blanquecinas en mucosa yugal. Cul es el
diagnstico ms probable?

Otits media aguda recidivante.


Otitis media serosa.
Otitis media crnica simple.
Otitis media crnica colesteatomatosa.
Mastoiditis.

167. Ante un traumatismo directo sobre el lado derecho de la cara tras el cual el paciente presenta
hematoma palpebral unilateral, diplopa en la
mirada vertical y dificultad para la apertura de
la boca. Cal de las siguientes afirmaciones es
cierta?.

1.
2.
3.
4.
5.

1. Estamos con gran probabilidad ante una


factura de tercio medio facial tipo Lefort I.
2. Se trata probablemente de una fractura bitomalar unilateral. El diagnstico se comprobara ideamente mediante TAC (Tomografa
axial comutarizada).
3. Se trata de una fractura de la base de crneo a
nivel del agujero carotdeo.
4. El diagnstico probable es la fractura con
dislocacin del cndilo mandibular.
5. Probablemente se asocia una fractura mandibular con una fractura del tercio medio facial
tipo Lefort I.

Pitiriasis versicolor.
Micosis fungoides.
Psoriasis en gotas.
Pitiriasis rosada de Gibert.
Liquen plano.

171. Enfermo de 60 aos que refiere hace 10 das la


aparicin de lesiones ampollosas en dorso de
manos despes de la exposicin solar. El cuadro
se acompaa de fragilidad cutnea. Histopatolgicamente existe una ampolla subepidrmica con
depsitos PAS positivos en y alrededor de los
vasos drmicos superficiales. El diagnstico ms
verosmil ser:

1.
2.
3.
4.
5.

168. Hombre de 35 aos que acude al servicio de


urgencias por dolor en su ojo derecho de 3 das
de evolucin. La exploracin biomicroscpica del
segmento anterior muestra, tras tincin con
fluorescena, una lcera corneal central en forma de dendrita. Cul es su diagnstico?

1.
2.
3.
4.
5.

Membrana epirretiniana macular.


Agujero macular.
Trombosis de la vena central de la retina.
Degeneracin macular senil.
Coriorretinopata serosa central.

Pnfigo benigno familiar.


Penfigoide.
Porfiria cutnea tarda.
Pnfigo vulgar.
Sndrome de la piel escaldada.

172. Si desea conocer la imprecisin esperada al estimar la media de una variable recurrir a:

1.
2.
3.
4.
5.

Queratitis herptica.
Abrasin corneal.
Queratitis fngica.
Queratitis bacteriana.
Crisis glaucomatocicltica.

el error estndar (tpico).


la desviacin estndar (tpica).
la varianza de la variable.
el rango intercuartlico.
el coeficiente de variacin.

173. En un contraste de hiptesis estadstico si la


hiptesis nula fuera cierta y se rechazara Cul
de las respuesta es CORRECTA?

169. Mujer de 75 aos diagnosticada hace tres aos


de drusas blandas en el fondo de ojo. Refiere
presentar, desde hace dos semanas, metamorfopsia y prdida visual importante en su ojo derecho que le impide leer. Seale el diagnstico ms
probable.

1.
2.
3.
4.
- 28 -

Se comete un error de tipo II.


Se toma una decisin correcta.
La potencia aumenta.
Se comete un error de tipo I.

5. Indica un tamao muestral excesivo

2. La confusin ocurre cuando una tercera variable se asocia tanto a la exposicin como al
desenlace.
3. Los anlisis estadsticos multivariantes reducen la confusin.
4. El diseo de los ensayos controlados con
distribucin aleatoria minimiza la confusin.
5. La medicin inexacta o sesgada de algunas
variables puede generar confusin.

174. Las curvas de Kaplan-Meier constituyen un


mtodo estadstico en:

1.
2.
3.
4.
5.

El anlisis de supervivencia.
La regresin de Poisson.
La regresin lineal.
La regresin logstica.
La determinacin de las odds ratios.

178. Cul de los siguientes diseos es un diseo experimental?

175. Se ha realizado un ensayo controlado y aletorizado en pacientes con hipercolesterolemia para


valorar la efectividad de un nuevo frmaco. La
variable final valorada es la presencia de infarto
de miocardio. El Riesgo Relativo (RR) de infarto
de miocardio en relacin con el frmaco habitual
es de 0,39 con un IC al 95% de 0,21 a 1,18. Ante
este resultado usted concluira que:

1.
2.
3.
4.
5.

Estudio de casos y controles.


Estudio de cohortes.
Estudio transversal.
Serie de casos.
Un ensayo clnico controlado.

179. Se ha realizado un estudio epidemiolgico con el


objetivo de dilucidar si existe asociacin entre la
administracin de una nueva vacuna antigripal y
la aparicin de sndrome de Guillain-Barr.
Para ello se recogieron los datos de todos los
sujetos vacunados en determinada rea geogrfica y mediante la conexion de estos datos con los
de nuevos diagnsticos de Sndrome de GuillainBarr registrados en los hospitales de ese mismo
rea, se compar la incidencia de Sndrome de
Guillain-Barre en sujetos expuestos y en no expuestos a la vacuna en una ventana temporal
definida. A qu tipo de diseo corresponde este
estudio?

1. Debera recomendarse la utilizacin del nuevo frmaco.


2. El nuevo frmaco reduce el riesgo relativo en
ms de un 60%.
3. El tamao muestral es muy elevado.
4. Existen diferencias estadsticamente significativas entre el efecto de los frmacos estudiados.
5. Los resultados revelan una precisin pobre.

176. El objetivo de una investigacin es determinar la


probabilidad de sufrir cirrosis heptica en funcin de la presencia o no de cinco variables:
sexo, edad, consumo de alcohol, consumo de
drogas y nivel de actividad fsica. La tcnica
estadstica ms adecuada para evaluar el objetivo propuesto es:

1. Estudio de cohortes.
2. Estudio de casos y controles anidado en una
cohorte.
3. Estudio de casos y controles de campo.
4. Estudio de corte transversal.
5. Estudio descriptivo.

1. El clculo de la matriz de correlacin entre


los factores.
2. El clculo de la regresin lineal.
3. El anlisis de componentes principales.
4. El clculo de la regresin logstica.
5. La prueba de "ji" cuadrado.

180. Para relacionar el alcohol con los accidentes de


trfico se procedi de la manera siguiente: Por
cada accidentado se seleccion un individuo no
accidentado que pasaba a la misma hora por una
carretera de caractersticas similares a la del
accidentado. Se les extrajo sangre para cuantificar la cifra de alcoholemia. Cul es el tipo de
diseo de estudio empleado?:

177. Cul de las siguientes afirmaciones acerca de la


confusin en los estudios epidemiolgicos es
falsa?

1.
2.
3.
4.
5.

1. Para "controlar" la confusin en un estudio


las variables que la producen deben conocerse obligatoriamente de antemano.
- 29 -

Estudio Transversal o de prevalencia.


Estudio de cohortes.
Estudio de casos y controles.
Ensayo clnico.
Estudio ecolgico.

Qu tipo de estudio sera el ms vlido y eficiente para estudiar la posible asociacin entre la
ocurrencia de espina bfida en el recin nacido y
la exposicin a diversos factores ambientales
durante el embarazo?

181. Se dice que un estudio observacional tiene validez interna:

1. Cuando se obtiene un resultado estadsticamente significativo.


2. Cuando el investigador ha controlado razonablemente las posibles fuentes de sesgo.
3. Cuando el investigador no ha cometido fraude y los resultados son autnticos.
4. Cuando se ha publicado en una revista con
alto factor de impacto.
5. Cuando el intervalo de confianza de la principal medida de efecto no incluye el valor
nulo.

1. Un estudio de cohorte prospectivo integrado


por mujeres que estn planificando el embarazo.
2. Un estudio de cohorte retrospectivo a travs
de las historias clnicas de mujeres que han
parido en los hospitales seleccionados.
3. Un estudio de casos y controles de base hospitalaria, tomando como casos a las madres
de nios que han nacido con espina bfida y
como controles a las madres de nios que
han nacido sin malformaciones, hacindoles
a ambos grupos un entrevista sobre exposiciones ambientales durante el embarazo.
4. Un estudio de casos y controles de base poblacional tomando como casos a las mujeres
expuestas a los factores ambientales de inters y como controles a las mujeres no expuestas.
5. Un estudio de corte transversal en la poblacin general de mujeres de edad comprendida entre los 20 y 40 aos.

182. Se ha realizado un estudio de cohorte retrospectivo para conocer si los pacientes que toman
antipsicticos presentan un mayor riesgo de
muerte sbita que la poblacin que no utiliza
antipsicticos. Una vez realizado el ajuste por
posibles factores de confusin se ha obtenido un
riesgo relativo de 2,39 (intervalo de confianza al
95% de 1,77-3,22). Cul es la interpretacin
ms correcta del resultado?

1. El resultado es compatible con un incremento


de riesgo asociado al uso de antipsicticos,
pero no es estadsticamente significativo.
2. El resultado sugiere que los antipsicticos
protegen frente al riesgo de muerte sbita.
3. El resultado no es interpretable porque no se
ha hecho una asignacin aleatoria de los
tratamientos.
4. Hay un incremento de riesgo pero es pequeo
e irrelevante desde un punto de vista clnico.
5. El resultado apoya la hiptesis de que el uso
de antipsicticos aumenta el riesgo de muerte
sbita.

185. Se desea conocer la utilidad de un medicamento


que ya est comercializado en una indicacin
diferente a la autorizada, dado que se han publicado algunos casos de pacientes tratados que
han mostrado resultados prometedores. Cual
de los siguientes diseos ofrecer la mayor evidencia cientfica?:

1. Ensayo clnico abierto, prospectivo, paralelo,


controlado con placebo en la nueva indicacin, con 60 pacientes por brazo de tratamiento.
2. Estudio observacional prospectivo en 300
pacientes seguidos durante un mnimo de 2
aos.
3. Meta-anlisis de los datos publicados hasta el
momento.
4. Estudio retrospectivo de casos y controles en
los ltimos 2 aos.
5. Ensayo clnico aleatorizado, doble ciego,
controlado con el tratamiento estndar de esa
indicacin en 60 pacientes por brazo de tratamiento.

183. En una comunidad se han detectado un cluster o


agregacin de casos de leucemia no atribuible a
la variabilidad habitual de la enfermedad. Cul
es el diseo ms adecuado para analizar su posible relacin con la exposicin a una fuente de
ondas electromagnticas?

1.
2.
3.
4.
5.

Un estudio transversal.
Un estudio de cohortes histricas.
Un estudio de cohortes.
Un estudio de casos y controles.
Un estudio ensayo clnico.

186. Va a realizar un ensayo clnico aleatorizado para


comparar la eficacia de dos tratamientos para
dejar de fumar. Como cree que el sexo puede
influir en el resultado, le gustara asegurar que
la proporcin de hombres y mujeres es similar
en los dos grupos de tratamiento. Que tipo de

184. Teniendo en cuenta que la prevalencia de espina


bfida es de 1 caso por cada 1.000 nacidos vivos
- 30 -

aleatorizacin utilizara?
190. Cundo vamos a realizar un ensayo clnico aleatorizado doble-ciego, en que consiste la tcnica
de doble simulacin?

1. Aleatorizacin por bloques.


2. Aleatorizacin estratificada.
3. No es necesaria la aleatorizacin si aseguramos el correcto enmascaramiento del tratamiento.
4. Aleatorizacin por centros.
5. Aleatorizacin simple.

1. Es un mtodo de aleatorizacin para asegurarse de que los factores de confusin se


distribuyen por igual entre los dos grupos de
tratamiento.
2. Consiste en que el estadstico que vaya a
analizar los datos no sepa el tratamiento que
ha correspondido a cada paciente.
3. Es un diseo cruzado en el que al paciente se
le administra cada vez un tratamiento sin que
sepa en cada momento que frmaco le ha
correspondido.
4. Es un mtodo para calcular el tamao de la
muestra en el que se simula el efecto esperando con cada uno de los dos tratamientos.
5. Es un mtodo de enmascaramiento donde
todos los individuos reciben uno de los tratamientos (experimental o control) junto con
el placebo del otro frmaco.

187. La autorizacin de la Agencia Espaola de Medicamentos es necesaria para el desarrollo de


cualquier ensayo clnico con medicamentos en un
centro sanitario. De que otro organismo tambin es imprescindible tener un informe favorable para desarrollar el ensayo clnico en el centro?

1. El Comit de tica Asistencial del centro


sanitario.
2. La Comisin Farmacoteraputica del centro
sanitario.
3. El Comit tico de Investigacin Clnica del
centro sanitario.
4. La Direccin de Investigacin del centro
sanitario.
5. La Unidad Central de Investigacin Clnica
en Ensayos Clnicos del centro sanitario.

191. Al realizar una serologa frente a Trypanosoma


cruzi en una comunidad de inmigrantes bolivianos con una prevalencia estimada del 30% de
enfermedad de Chagas, respecto a otra comunidad de inmigrantes ecuatorianos con una prevalencia estimada de enfermedad de Chagas del
5%:

188. Un ensayo clnico "abierto" significa:

1.
2.
3.
4.
5.

1.
2.
3.
4.

Aumenta la sensibilidad.
Disminuye la especificidad.
Aumenta el valor predictivo positivo.
Aumenta en valor absoluto los falsos positivos.
5. Aumenta la validez interna de la serologa.

No requiere ingreso.
Pueden incluirse sujetos en el ensayo.
No existe grupo control.
No enmascarado.
Sus resultados son extrapolables a la poblacin general.

192. Una prueba de laboratorio aparece alterada en 8


de cada 10 individuos que padecen una determinada enfermedad crnica mientras se obtienen
valores normales en 8 de cada 10 individuos que
no sufren dicha enfermedad. Si se aplica la prueba a una poblacin de 100 individuos de los cuales 20 sufren la enfermedad cul ser el valor
predictivo positivo de la prueba?

189. Se est planificando un ensayo clnico en Fase III


para evaluar la eficacia, en trminos de erradicacin microbiolgica, de una nueva fluorquinolona en pacientes con infeccin del tracto urinario. Cul de los siguientes diseos es el ms
apropiado?

1. Diseo factorial.
2. Paralelo, abierto, controlado con placebo.
3. Paralelo, aleatorizado, doble ciego, controlado con otro antibitico activo frente a Gram
negativos.
4. Cruzado, aleatorizado, doble ciego, controlado con otro antibitico activo frente a Gram
negativos.
5. Cruzado, aleatorizado, doble ciego, controlado con placebo.

1.
2.
3.
4.
5.

- 31 -

40%
50%
60%
70%
80%

193. Si consideramos una cifra de 24mmHg de tensin intraocular medida por tonometra ocular
como criterio diagnstico de glaucoma en lugar
de 20mmHg:

1.
2.
3.
4.
5.

segn pauta estandard frente hepatitis B hace 4


aos. Sera recomendable:

1.
2.
3.
4.
5.

Aumenta el nmero de verdaderos negativos.


Disminuye el nmero de falsos negativos.
Aumenta el nmero de verdaderos positivos.
Aumenta el nmero de falsos positivos.
Aumenta la sensibilidad y disminuye la especificidad.

198. Nos consulta una mujer de 84 aos, por insomnio de conciliacin. Tras fracasar a medidas de
higiene del sueo, se decide iniciar tratamiento
farmacolgico. Cul de los siguientes frmacos
seleccionara para la paciente?

194. La sensibilidad de una prueba diagnstica mide:

1. La proporcin de casos de pacientes sin la


enfermedad que presentan un resultado negativo de la prueba diagnstica.
2. La proporcin de casos de pacientes sin la
enfermedad que presentan un resultado positivo de la prueba diagnstica.
3. La proporcin de pacientes que se someten a
la prueba que tienen la enfermedad.
4. La proporcin de casos de enfermos con
resultado positivo de la prueba diagnstica.
5. La proporcin de casos con resultado positivo de la prueba que son verdaderamente
enfermos.

1.
2.
3.
4.
5.

Diacepam.
Lormetacepam.
Fenobarbital.
Clordiacepxido.
Cloracepato.

199. De cul de los siguientes frmacos no se suele


realizar la determinacin de las concentraciones
plasmticas en la prctica clnica?

1.
2.
3.
4.
5.

195. En qu tipo de evaluacin econmica se tienen


en cuenta las "preferencias de los pacientes"?

1.
2.
3.
4.
5.

Reiniciar pauta vacunal (0-1-6 meses).


Reiniciar pauta vacunal (0-1-2-12 meses).
No revacunar.
Darle una dosis de recuerdo o booster.
Darle Immunoglobulina anti-hepatitis B y
reiniciar pauta vacunal (0-1-2-12 meses).

Anlisis coste-beneficio.
Anlisis de minimizacin de costes.
Anlisis coste-efectividad.
Anlisis coste-utilidad.
Estudios de coste de la enfermedad.

Acenocumarol.
Gentamicina.
Digoxina.
Fenitona.
Litio.

200. Cuando se realiza un estudio de bioequivalencia


para evaluar si un frmaco genrico es bioequivalente con el producto original, Cul es el
criterio principal utilizado habitualmente por las
agencias reguladoras (Agencia Espaola y Agencia Europea) para su autorizacin?

196. Cul de las siguientes herramientas resulta


menos til en la toma de decisiones sobre estrategias teraputicas desde el enfoque de la gestin
clnica?

1. La cantidad de principio activo y los excipientes deben ser los mismos en el genrico y
en el original.
2. El cociente de las medias de la formulacin
test con respecto a la formulacin de referencia para AUC y Cmax debe ser del 100%.
3. El intervalo de confianza del 90% del cociente de las medias de la formulacin test con
respecto a la formulacin de referencia para
AUC y Cmax debe estar incluido entre los
lmites 80-125%.
4. El medicamento genrico debe contener al
menos el 80% del principio activo que contiene el producto de referencia.

1. La medicina basada en la evidencia.


2. Las guas de prctica clnica.
3. Las bsquedas sistemticas de informacin:
Tripdatabase, Pubmed, Cochrane, ...
4. Los informes de evaluacin de tecnologas
sanitarias.
5. La experiencia profesional individual.

197. Mdico traumatlogo de 40 aos que en una


serologa rutinaria se le detecta niveles de antiHBs de 30 UI/l. Refiere vacunacin completa
- 32 -

5. El cociente del AUC de la formulacin test


con respecto al AUC de la formulacin de
referencia para todos los sujetos debe estar
incluido entre los lmites 80-125%.

ms frecuente de ITU en las gestantes?

1.
2.
3.
4.
5.

201. Un nio de 2 aos de edad es trado al Servicio


de Urgencias porque se ha tomado el contenido
del frasco de Apiretal (paracetamol) que estaba hacia la mitad, hace aproximadamente 30
min. El nio tiene buen estado general y la exploracin fsica es anodina. Se inicia la administracin de carbn activado y se mantiene en
observacin. Cul es la actitud a seguir a partir
de ese momento?

204. Acinetobacter baumannii es un microorganismo


asociado a infeccin nosocomial que se caracteriza por:

1. Ser un bacilo Gram positivo fermentador de


azcares
2. Ser un parsito obligado de clulas de rin
humanas
3. Su facilidad para mantenerse y diseminarse
en el hospital exclusivamente a travs de las
conducciones de aire.
4. Ser sensible a todos los antibiticos betalactmicos
5. Su facilidad para adquirir resistencia a los
antibiticos unido a su capacidad de generar
infecciones en pacientes inmunocomprometidos

1. Solicitar las transaminasas hepticas en sangre y si no estn alteradas darle el alta.


2. Mantenerlo en observacin hasta que hayan
pasado 4 h desde la ingesta, extraer sangre
para determinar la concentracin de paracetamol y comprobar que se dispone de Nacetilcisteina intravenosa por si hubiera que
administrarla.
3. Mantenerlo en observacin hasta que hayan
pasado 4 h desde la ingesta, extraer sangre
para determinar la concentracin de paracetamol y comprobar que se dispone de Protamina que es el antdoto necesario.
4. Mantenerlo en observacin hasta que hayan
pasado 6 h desde la ingesta, extraer sangre
para determinar la concentracin de paracetamol y comprobar que se dispone de Atropina que es el antdoto necesario.
5. Ponerse en contacto con el Coordinador de
transplantes por si fuera necesario el transplante heptico.

205. Cal de las siguientes afirmaciones referidas a


los virus herpes humanos es correcta?.

1. Son virus RNA de simetra icosadrica y


desnudos (sin envoltura).
2. Los mecanismos involucarados en la patogenia de las infecciones causadas por los virus
del herpes simple tipos 1 y 2 son muy diferentes.
3. La queratitis herptica casi siempre afecta a
ambos ojos.
4. El virus de Epstein Barr presenta una relacin etiolgica con el linfoma endmico de
Burkitt, la enfermedad de Hodgkin y el carcinoma nasofarngeo.
5. La va frecuente de transmisin de citomegalovirus es la area.

202. Cul de los siguientes antiepilpticos se espera


que produzca un menor nmero de interacciones
farmacolgicas?

1.
2.
3.
4.
5.

Escherichia coli.
Enterococcus faecalis.
Streptococcus agalactiae.
Proteus mirabilis.
Staphylococcus saprophyticus.

Fenitona.
Acido Valproico.
Levetiracetam.
Carbamazepina.
Oxcarbazepina.

206. Paciente de 15 aos que acude por la noche al


servicio de Urgencias con un cuadro de nuseas,
vmitos, molestias abdominales y diarrea acuosa
(5 episodios) que se inici hace unas 5-6 horas.
No presenta fiebre. En la anamnesis refiere que
su madre y otros 3 familiares padecen sntomas
similares. Todos los afectados comieron durante
una excursin ensaladilla con mahonesa y hamburguesas de pollo. Cul es el agente etiolgico
ms probable de la infeccin?

203. Gestante de 27 aos, 30 semanas de gestacin.


Acude a Urgencias por notar desde ayer dolor en
regin lumbar izquierda y disuria. No tiene sensacin febril. Refiere infecciones del tracto urinario (ITU) reiteradas. En el anlisis de orina se
observan: Hb 3+, leucocitos 3+, nitritos 2+. Sedimento: 15-20 leucocitos/campo y 5-10 hemates/campo. Ante la conveniencia de instaurar un
tratamiento antibitico emprico, cul de los
siguientes microorganismos es el responsable
- 33 -

1.
2.
3.
4.
5.

Staphylococcus aureus.
Salmonella.
Escherichia coli enterotoxignica.
Rotavirus.
Campylobacter jejunii.

1.
2.
3.
4.
5.

207. Debido a las caractersticas de su ciclo de replicacin, Cul de los siguientes virus puede ser
erradicado del organismo tras ser sometido a
tratamiento antiviral con agentes de accin directa?

211. El triangulo de Koch se encuentra en la aurcula


derecha concretamente en la porcin inferior
del tabique interauricular siendo uno de sus
lmites la valva septal de la vlvula tricspide; es
importante su conocimiento ya que en l se encuentra un elemento del sistema de conduccin
cardiaca y cuya lesin en ciruga cardiaca provocara alteraciones graves en el sistema de conduccin. Qu estructura de las citadas a continuacin se encuentra localizada en este tringulo?

1. El virus de la inmunodeficiencia humana


(VIH).
2. El virus del herpes simple (VHS).
3. El virus de Epstein-Barr (VEB).
4. Citomegalovirus (CMV).
5. El virus de la hepatis C (VHC).

208. Un individuo presenta "debilidad muscular"


(hemiparesia espstica) de ambas extremidades
derechas, con hiperreflexia y signo de Babinski,
junto a una "parlisis flccida facial" de la
hemicara izquierda, con incapacidad para cerrar
el ojo izquierdo o de retraer el lado izquierdo de
la boca, adems de otras alteraciones. Por los
datos descritos se trata de una alteracin que
afecta, entre otros elementos, a los fascculos
motores: crtico-espinal y crtico-nuclear, pero
a qu nivel del neuroeje localizara la lesin?

1. Ndulo sinusal o sinoauricular o de KeithFlack


2. Fascculo aurculo ventricular o haz de His
3. Rama derecha del haz de His
4. Ndulo aurculo ventricular o ndulo de
Tawara
5. Haz de Bachmann

212. Hombre de 42 aos de edad, con antecedentes


familiares de cncer colo-rectal y cncer edometrial de su madre y su hermana, presenta en una
colonoscopia plipos dispersos (4-5 plipos), y en
ngulo heptico, una masa tumoral ulcerosa y de
apariencia infiltrativa, que se biopsia. Histolgicamente se correspode con un adenocarcinona
moderamadente diferenciado de tipo mucinoso.
Los plipos resecados son adenomas tubulares
con displasia leve. Ante qu sndorme hereditario y mutacin gentica es ms probable que nos
encontremos?

1. A nivel del rea 4 de Brodmann de la corteza


cerebral del lado derecho.
2. En la cpsula interna, brazo posterior del
lado derecho.
3. En el pednculo cerebral izquierdo.
4. En la porcin medial de la protuberancia
caudal del lado izquierdo.
5. En el bulbo raqudeo antes de la decusacin
del fascculo crtico-espinal derecho.

209.

Ectodermo ventral
Esclerotomo
Mesodermo intermedio
Celoma intraembrionario
Endodermo

1. Poliposis adenomatosa familiar. Mutacin de


gen APC.
2. Sndrome de Li-Fraumeni. Mutacin en
TP53.
3. Cncer de colon no polipsico (sndrome de
Lynch). Mutacin de MSH2 y MLH-1.
4. Sndrome de Cowden. Mutacin en PTEN.
5. Sndrome de Gardner. Mutacin del gen
APC.

El mecanismo de pinza de la mano requiere la


integridad de:

1. El nervio radial.
2. El nervio cubital.
3. El nervio cubital con la accin combinada del
nervio radial.
4. El nervio mediano.
5. El nervio mediano con la accin combinada
del nervio radial.

213. Una glndula tiroides difusamente aumentada


muestra al examen microscpico un amplio infiltrado inflamatorio mononuclear (linfocitos pequeos y clulas plasmticas) centros germinales bien desarrollados y folculos tiroideos atrfi-

210. El aparato urogenital es un derivado del:


- 34 -

cos o recubiertos por clulas de Hrthle. Cul


es el diagnstico?:

1.
2.
3.
4.
5.

217. En cul de las siguientes patologas se incluye el


tratamiento con gammaglobulinas humanas?

Tiroiditis granulomatosa.
Adenoma de clulas de Hrthle (oxiflico).
Enfermedad de Graves.
Tiroiditis de Hashimoto.
Tiroiditis de Riedel.

1.
2.
3.
4.
5.

214. Si el informe anatomopatolgico del la biopsia de


una adenopata del mediastino indica la presencia de mltiples granulomas no caseificantes con
abundantes macrfagos activados, cul de las
siguientes enfermedades es ms probable?

Alergia al veneno de abeja.


Rechazo cardiaco.
Angioedema hereditario.
Prpura trombocitopnica idioptica.
Osteoartritis.

218. El sistema plasmtico de complemento es uno de


los principales mecanismos efectores de la respuesta inmune. Seale la opcin correcta entre
los siguientes enunciados referentes a dicho sistema.

1. Sarcoidosis.
2. Sepsis de origen pulmonar por bacterias
grampositivas.
3. Histiocitosis de clulas de Langerhans.
4. Tuberculosis pulmonar.
5. Metstasis de timoma invasivo.

1. Las deficiencias de los componentes finales


del sistema se manifiestan fundamentalmente
por enfermedades de tipo autoinmune debido
al depsito de inmunocomplejos.
2. La deficiencia del componente C2 provoca
fundamentalmente infecciones de repeticin
por bacterias del grupo Neisseria.
3. El diagnstico de edema angioneurtico
hereditario se establece mediante la cuantificacin de los niveles sricos circulantes y/o
el estudio de la actividad funcional del C1inhibidor.
4. El estudio de la actividad hemoltica total del
sistema de complemento no aporta datos de
inters en el estudio de las deficiencias de
dicho sistema.
5. Las infecciones recurrentes por estafilococos
y estreptococos son habituales en el deficit
del factor H.

215. En una caracterstica particular del sistema


inmune innato:

1. Su especificidad inmunologca.
2. La induccin de memoria inmunitaria permanente.
3. El reconocimento de patrones moleculares
vinculados a patgenos (PAMP) por protenas de la familia Toll (TLR).
4. La no intervencin de clulas presentadoras
de antgeno.
5. La participacin de linfocitos B y linfocitos
T.

219. En qu gen es ms frecuente una mutacin en


relacin con melanoma maligno?
216. Qu afirmacin es CORRECTA con respecto al
reconocimiento de antgenos por linfocitos T?
1.
2.
3.
4.
5.

1. Los linfocitos T reconocen determinantes


antignicos conformacionales o no lineales.
2. Los linfocitos T reconocen antgenos solubles y en forma nativa (no desnaturalizada o
no procesada) tal y como se presentan en la
naturaleza).
3. Los linfocitos T CD8 + reconocen pptidos
exgenos asociados a moleculas MHC de
clase II.
4. Los linfocitos T CD4 + reconocen pptidos
citoslicos (endgenos) asociados a molculas MHC clase I.
5. Los linfocitos T reconocen antgenos peptdicos asociados a molculas MHC (Major
Histocompatibility Complex) propias.

BRAF.
RET.
EGFR.
HER2/neu.
TP53.

220. Un hombre presenta una delecin parcial en el


gen de la distrofina (cromosoma Xp21) que le
ocasiona la semiologa propia de la distrofia
muscular de Becker. Acude a la consulta de gentica con su esposa, para valorar los riesgos de
transmisin de la enfermedad. Qu informacin correcta ser proporcionada en el transcurso del consejo gentico?

- 35 -

1. Segn la herencia autosmica dominante, la


mitad de sus hijos heredarn la enfermedad y
sin distincin de sexos.
2. Sus hijas no heredarn la enfermedad, pero
todos sus futuros hijos varones sern portadores y pueden transmitir la mutacin al
50%.
3. Sus hijos varones no heredarn la enfermedad, pero todas sus futuras hijas sern portadoras y pueden transmitir la mutacin al
50%.
4. No hay riesgo: la herencia de la enfermedad
es del tipo mitocondrial, nunca transmitida
por los varones.
5. Segn la herencia autosmica recesiva, un
25% de sus hijos manifestarn la enfermedad
en la infancia, sin distincin de sexos.

223. La reabsorcin de sodio en la nefrona distal


aumentar cuando se produzca:

1. Un aumento de la osmolaridad plasmtica.


2. Un aumento del volumen de plasma.
3. Un incremento de la concentracin de sodio
en plasma.
4. Un aumento de la presin arterial media.
5. Un incremento de la concentracin de potasio en plasma.

224. Conociendo la funcin de las estructuras relacionadas con la percepcin del dolor y asumiendo que la estimulacin elctrica puede bloquear
la transmisin del dolor, qu estructura no
debera ser estimulada elctricamente para el
tratamiento del dolor?

221. Una nia de 8 aos (caso ndice) est diagnosticada clnicamente como afecta de neurofibromatosis tipo 1 (NF1) o enfermedad de Von Recklinghausen con mltiples neurofibromas, manchas caf con leche y ndulos de Lisch. Su padre
(no diagnosticado de NF1) falleci por accidente
de circulacin a los 38 aos. La madre presenta
a la exploracin dos manchas caf con leche y
acude a la consulta de consejo gentico con su
nueva pareja donde se plantea un diagnstico
gentico pre implantacin (DGP). Est indicado
en este caso un DGP?

1.
2.
3.
4.
5.

Corteza sensorial primaria.


Ncleos intralaminares del tlamo.
Sustancia gris periacueductal/periventricular.
Columnas dorsales de la mdula espinal.
Estimulacin elctrica transcutnea de aferencias primarias de bajo umbral.

225. Que tipo de fibras vegetativas son las que inervan las glndulas sudorparas y los msculos
piloerectores?

1. S, al tener la madre 2 manchas caf con


leche es portadora y el DGP est indicado
con estos datos.
2. Est indicado tras detectar la mutacin causante en el caso ndice y eventualmente en su
madre.
3. No est indicado pues la NF1 responde a
mutaciones en el gen neurofibromn
(17q11.2), con herencia recesiva.
4. No, dos manchas caf con leche no son diagnsticas y su nueva pareja es muy improbable que sea portador (la NF1 es una enfermedad poco frecuente).
5. Est indicado con estos datos un DGP consistente en seleccionar embriones in vitro, para
implantar en el tero materno aquellos sin la
mutacin.

1.
2.
3.
4.
5.

Fibras simpticas adrenrgicas.


Fibras simpticas colinrgicas.
Fibras parasimpticas adrenrgicas.
Fibras parasimpticas colinrgicas.
Las glndulas sudorparas no poseen invervacin, siendo controladas nicamente por
factores humorales.

226. La intervencin quirrgica ms adecuada para


un paciente con megacolon txico en el contexto
de una colitis ulcerosa es:

1. Una hemicolectoma izquierda.


2. Una colectoma total con anastomosis ileorrectal.
3. Una colectoma total con ileostoma terminal.
4. Una proctocolectoma total y anastomosis
ileoanal con reservorio ileal.
5. Una ileostoma terminal manteniendo el
colon en reposo.

222. La precarga cardiaca aumenta cuando se produce:

1. Una disminucin del flujo sanguneo coronario.


2. Una disminucin de la complianza venosa.
3. Un aumento de la contractilidad miocrdica.
4. Una disminucin de la postcarga.
5. Un aumento de la permeabilidad capilar.
- 36 -

5. Infeccin por Gonococo.

227. La expresin clnica ms frecuente de la hipertensin arterial esencial es:

1.
2.
3.
4.
5.

231. Un lactante de 7 meses acude a urgencias derivado por su pediatra de zona por sospecha de
Enfermedad de Kawasaki. Refiere fiebre de
hasta 39,5 C de 5 das de evolucin que no cede
a pesar de tratamiento con amoxicilina pautado
hace 72 horas por sospecha de faringoamigdalitis aguda. Durante las ltimas 24 horas asocia
exantema eritematoso no pruriginoso de evolucin cefalo-caudal y durante los ltimos das
llama la atencin a sus padres la tos y una intensa conjuntivitis. No refieren rinorrea. A la exploracin clnica presenta subcrepitantes en ambas
bases pulmonares sin taquipnea ni signos de
dificultad respiratoria. A la exploracin orofarngea se evidencian manchas blanquecinas en
mucosa yugal. Cul es el diagnstico ms probable?

Ninguna.
Cefalea.
Astenia.
Disnea.
Epistaxis.

228. En un paciente en estudio por sospecha de fibrosis pulmonar idioptica cul de los siguientes
sntomas o signos debe hacernos sospechar un
diagnstico alternativo?

1.
2.
3.
4.

Presencia de sntomas sistmicos.


Acropaquias.
Disnea de esfuerzo progresiva.
Crepitantes finos en la auscultacin pulmonar.
5. Edad superior a 50 aos.

1.
2.
3.
4.
5.

229. Hombre de 54 aos, con antecedentes personales


de DM, HTA e insuficiencia renal crnica leve,
que presenta somnolencia y hemiparesia derecha. En la analtica: creatinina de 2.3 mg/dl, Hb
10.3 g/dl y plaquetas 20.000 mm3, con coagulacin normal. En TAC craneal: lesiones isqumicas con necrosis microhemorrgicas. Posteriormente comienza con deterioro progresivo del
nivel de conciencia y aumento de Cr, LDH y
bilirrubina no conjugada. Se realiza Coombs
directo que es negativo. Esquistocitos en frotis.
Ante la sospecha diagnstica la conducta a seguir es:

232. Cul sera el diseo ms adecuado para evaluar


la eficacia de un nuevo tratamiento para detener
la progresin de la enfermedad de Alzheimer
leve-moderada?

1. Ensayo clnico aleatorizado, paralelo, doble


ciego, controlado con placebo, de 2 aos de
tratamiento
2. Ensayo clnico con control histrico de 4
aos de tratamiento
3. Estudio retrospectivo de casos y controles
4. Estudio observacional de seguimiento prospectivo durante 5 aos
5. Ensayo clnico aleatorizado, abierto, comparado con el tratamiento estndar, de 6 meses
de tratamiento

1. Nuevo TAC craneal ante la sospecha de


transformacin hemorrgica de ictus isqumico.
2. Iniciar plasmafresis urgente.
3. Inicio de tratamiento con glucocorticoides a
dosis de 1mg/kg/da.
4. Buscar causa desencadenante de cuadro de
CID.
5. Sospecha de Mieloma Mltiple: aspirado de
mdula sea.

233. En una paciente de 35 aos con un sndrome


depresivo en tratamiento con inhibidores de la
recaptacin de la serotonina est contraindicado
el empleo de uno de los siguientes antimicrobianos:

230. Paciente de 40 aos, que acude a urgencias por


estomatitis aftosa, conjuntivitis, uretritis, balanitis y dolor articular. Cal de los siguientes diagnsticos es el ms probable?.

1.
2.
3.
4.

Enfermedad de Kawasaki.
Escarlatina.
Reaccin alrgica a la amoxicilina.
Mononucleosis infecciosa.
Sarampin.

1.
2.
3.
4.
5.

Behet.
Sndrome de artritis reactiva.
Sndrome de Sweet.
Dficit de vitamina A.
- 37 -

Doxiciclina.
Amoxicilina-Clavulnico.
Daptomicina.
Linezolid.
Vancomicina.

234. Cmo actan la Cianamida o el Disulfiram en


un paciente alcohlico?

1. Reducen el efecto del OH en el organismo,


disminuyendo el posible efecto positivo de
un nuevo consumo
2. Incrementan el efecto txico del OH, desmotivando al paciente de cara a nuevos consumos
3. Anulan el efecto del OH en el organismo,
actuando como antagonistas
4. Por su efecto estabilizador del nimo posibilitan un mayor control del consumo
5. Ocupan los receptores GABA, reduciendo el
"craving"

235. Paciente de 60 aos diagnosticada de neoplasia


de mama hace 10 aos. Realiz tratamiento con
radioquimioterapia y posteriormente tratamiento hormonal durante 5 aos. Un estudio con
gammagrafa sea realizado por dolores seos
politpicos demostr la presencia de metstasis
seas. Actualmente est en tratamiento con
opioides menores y AINES con buen control del
dolor. Acude a consulta por cefalea que no cede
con la actual analgesia realizando una TC cerebral que muestra imgenes compatibles con
metstasis cerebrales. En relacin al tratamiento
del dolor indique la CORRECTA:
1. Se debe cambiar a opioides mayores.
2. Se deben administrar las dosis extras de
opioides que sean necesarias.
3. Se deben aadir corticoides.
4. Se debe cambiar a un opioide mayor y mantener los AINES.
5. Se debe ingresar al paciente para tratamiento
endovenoso con opioide mayor.

- 38 -

Vous aimerez peut-être aussi